MCAT Chemistry Flashcards

You may prefer our related Brainscape-certified flashcards:
1
Q

Many double-displacement reactions are enzyme-catalyzed via the “ping-pong” mechanism, so called because the reactants appear to bounce off the enzyme like a ping pong ball. These reactions typically have two reactants and two products. In a third-order reaction involving two reactants and two products, doubling the concentration of the first reactant causes the rate to increase by a factor of 2. What will happen to the rate of this reaction if the concentration of the second reactant is cut in half?

A) It will increase by a factor of 2
B) It will increase by a factor of 4
C) It will decrease by a factor of 2
D) It will decrease by a factor of 4

A

D

Based on the information given in the question, the rate is first-order with respect to the concentration of the first reactant; when the concentration of that reactant doubles, the rate also doubles. Because the reaction is third-order, the sum of the exponents in the rate law must be equal to 3. Therefore, the reaction order with respect to the other reactant must be 3-1=2. If the concentration of this second reactant is multiplied by 1/2, the rate will be multiplied by (1/2)^2 = 1/4.

How well did you know this?
1
Not at all
2
3
4
5
Perfectly
2
Q

For the most part, biological pH is slightly basic. However, the stomach is very acidic, and its contents must be swiftly neutralized by basic bicarbonate ions upon entering the small intestine, which has a basic pH. How many liters of 2M Ba(OH)2 are needed to titrate a 4L solution of 6M H3PO4?

A) 1.33L
B) 12L
C) 18L
D) 56L

A

C

Use the equivalence point equation:

(N1)(V1) = (N2)(V2)

Ba(OH)2 can dissociate to give two hydroxide ions, so its normality is 2M x 2 = 4N. H3PO4 can dissociate to give three hydronium ions, so its normality is 6M x 3 = 18N. Plugging into the equation, we get (18N)(4L) = (4N)(V2). Therefore, V2 is 18L.

How well did you know this?
1
Not at all
2
3
4
5
Perfectly
3
Q

The body’s pH is tightly regulated because specific enzymes function best within a narrow pH range. What is the approximate pH of a 1.2 x 10^-5 M aqueous solution of NaOH?

A) 4.92
B) 7.5
C) 9.08
D) 12.45

A

C

NaOH is a strong base; as such, there will be 1.2 x 10^-5 M OH- in solution. Based on this information alone, the pOH must be between 4 and 5, and the pH must be between 9 and 10. Using the shortcut, pOH ~5 - 0.12 = 4.88. pH = 14 - pOH = 9.12 (actual = 9.08).

How well did you know this?
1
Not at all
2
3
4
5
Perfectly
4
Q

For some compounds, the empirical and molecular formula are identical. H2O is an example of this. Which of the following molecules does not have the empirical formula CH?

A) Benzene
B) Ethyne
C) Acetylene
D) Toluene
E) None of the above
A

D

Benzene has a molecular formula of C6H6, making its empirical formula CH. Ethyne has a molecular formula of C2H2, making its empirical formula CH as well. Acetylene is just another name for ethyne. Toluene has a molecular formula of C7H8, which is also its empirical formula, not CH.

How well did you know this?
1
Not at all
2
3
4
5
Perfectly
5
Q

Which of the following compounds has the highest boiling point?

A) Heptane
B) Octane
C) 2-methylheptane
D) 2,2-dimethylhexane

A

B

The boiling point of hydrocarbons increases as the number of carbons increases. Therefore, heptane has a higher boiling point than hexane, which has a higher boiling point than pentane. This is because the intermolecular forces between molecules keep the hydrocarbons in solution, requiring more heat to overcome these forces and become a gas. In the case of branched molecules, the hydrocarbons are not able to “stack” as easily as straight-chain hydrocarbons. This reduces intermolecular interactions and, subsequently, the boiling point. Even though octane, 2,2-dimethylhexane, and 2-methylheptane have eight carbons, octane has the highest boiling point.

How well did you know this?
1
Not at all
2
3
4
5
Perfectly
6
Q

Which of the following best represents the relationship among the boiling points in FBr, argon, and Cl2?

A) Ar < FBr < Cl2
B) FBr < Cl2 < Ar
C) Cl2 < Ar < FBr
D) Ar < Cl2 < FBr

A

D

The order of boiling points corresponds to the strength of intermolecular forces. Argon has the weakest intermolecular forces: as a neutral atom, it is less polarizable than the larger Cl2. The latter has no dipole moment, so it exerts weaker forces than FBr.

How well did you know this?
1
Not at all
2
3
4
5
Perfectly
7
Q

Which of the following is closest to the bond angle of an sp2-hybridized carbon atom?

A) 180•
B) 140•
C) 120•
D) 110•

A

C

An sp2-hybridized carbon has a trigonal planar geometry. This means that the bond angle will be 120•. An sp-hybridized carbon has a linear geometry, meaning a bond angle of 180•. Finally, an sp3-hybridized carbon has a tetrahedral geometry, with bond angles of 109.5•.

How well did you know this?
1
Not at all
2
3
4
5
Perfectly
8
Q

Which of the following equations could be possibly used to determine the rate of diffusion of a gas in an enclosed flask once the valve is opened?

A) PE = mgh
B) KE = (1/2)mv^2
C) F = kQq/r^2
D) PV = nRT

A

B

The rate of expansion (or diffusion) is directly proportional to the mean velocity of the gas molecules. The only equation that has a term for velocity is B.

How well did you know this?
1
Not at all
2
3
4
5
Perfectly
9
Q

Which of the following is NOT characteristic of hydrogen bonding?

A) The hydrogen atom involved must be covalently bonded to a very electronegative atom
B) The hydrogen bonds are typically weaker than ionic or covalent bonds
C) The other atom involved in the hydrogen bond (not the hydrogen atom) must be covalently bonded to a hydrogen atom
D) The other atom involved in the hydrogen bond (not the hydrogen atom) must possess at least one lone pair of electrons

A

C

Answer choice C is not necessary for the formation of a hydrogen bond. A classic example is that of ethers. Ethers are water soluble because the oxygen in ether, which is covalently bonded to two carbons, can hydrogen bond with the hydrogen in H2O. Note regarding answer choice C: the 3-D shapes of many proteins and DNA is due in part to intramolecular H-bonds.

How well did you know this?
1
Not at all
2
3
4
5
Perfectly
10
Q

Which of the following is the correct way to dispose of excess reagent?

A) Add the excess back into the reaction
B) Return excess to the stock bottle
C) Flush down the sink with lots of water
D) Dispose in the liquid container

A

D

When dealing with excess reagent, you should never pour it back into the stock solution to avoid contamination. Unless told otherwise, it is best to only drain water in the sinks of a chemistry lab. If the reagent is toxic or flammable, leaving it in an open container is not a safe practice. Therefore, the proper way to dispose of excess chemicals is to put it in a clearly marked chemical waste container.

How well did you know this?
1
Not at all
2
3
4
5
Perfectly
11
Q

The isoelectric point of glycine is 6.0. When glycine is in a buffer with a pH of 6.0, which form predominates?

A) H3N+-CH2-COO-
B) H2N-CH2-COOH
C) H3N+-CH2-COOH
D) H2N-CH2-COO-

A

A

Recall that you can use isoelectric point (PI) to predict an amino acid’s charge at any given pH. Specifically, if PI < pH, then this means that pH is high – which means that [H+] is low, and charge is negative. Likewise, if PI > pH, then this means that pH is lower — which means [H+] is high and charge is positive. In this case, PI = pH, which means that charge is zero (the amino acid is neutral). Thus, the answer is A since this is the amino acid where both charges cancel each other out, leaving a neutral zwitterion glycine.

How well did you know this?
1
Not at all
2
3
4
5
Perfectly
12
Q

When metallic substances lose electrons, they are trying to achieve the electron configuration of what?

A) The metal immediately before it on the periodic table
B) The metal immediately after it on the periodic table
C) The nearest halogen
D) The previous noble gas

A

D

A metal prefers to lose electrons so that it has the configuration of 8 around its valence shell so that it is equivalent to the stable noble gas previous to it.

How well did you know this?
1
Not at all
2
3
4
5
Perfectly
13
Q

Which of the following acids have the largest pKa?

A) CH3CO2H
B) FCH2CO2H
C) CF3CO2H
D) Cl3CCO2H

A

A

Recall that pKa = -log Ka so their relative values, because of the negative sign in the equation, are always opposite to each other. Thus in an acid base equilibrium, AH A- + H+, the largest pKa value is associated with the weakest acid. The unhalogenated carboxylic acid A is the weakest acid (acetic acid is the IUPAC name; ethanoic acid is the systematic way to name it). In the other cases, the electron withdrawing halogen substituent stabilize the carboxylate conjugate base (anion) making the parent acid more acidic.

How well did you know this?
1
Not at all
2
3
4
5
Perfectly
14
Q

A student creates a 1M citrate (C3H5O(COO)3 3-) buffer solution at pH 5.0. He then dilutes the solution to 200 mM, and adds it to his experiment. What is the pH of the 200 mM buffer solution?

A) Between pH 5 and pH 7
B) Greater than pH 7
C) Less than pH 5
D) The pH will remain close to 5

A

D

Buffers are designed to maintain pH at a certain level, and operate through the complementation of a conjugate acid and base. As long as the ratios of acid and base remain the same in the buffer system, and they will no matter what the dilution, as long as the dilution is not very very small (i.e. 1 mM or less), the pH will remain the same.

How well did you know this?
1
Not at all
2
3
4
5
Perfectly
15
Q

The following reaction was carried out in a closed system: N2(g) + O2(g) 2NO(g)
Hrxn = -ve value

Which of the following will favor the production of NO?

A) Decreased pressure
B) A catalyst
C) Increased pressure
D) Decreased temperature

A

D

Since the reaction is exothermic (negative enthalpy), heat can be considered a product in this reaction. A decrease in temperature will result in the equilibrium shifting to the right to bring heat to the system resulting in an increase in NO production, in accordance with Le Chatelier’s principle. A change in overall pressure of the system will not affect the equilibrium because there is an equal amount of moles of gas on both sides of the equation providing an even pressure. A catalyst affects the rate of a reaction not thermodynamic properties such as equilibrium.

How well did you know this?
1
Not at all
2
3
4
5
Perfectly
16
Q

Consider the following reaction:

NH3(aq) + H2O(l) -> NH4+(aq) + OH-(aq)
Keq = 1.81 x 10^-5

Which of the following can be concluded from the chemical equilibrium constant?

A) The reaction is slow to reach chemical equilibrium
B) The equilibrium mixture has more reactants than products
C) The equilibrium mixture has more products than reactants
D) The equilibrium mixture evaporates quickly leaving only products

A

B

The chemical equilibrium constant is calculated by dividing the concentration of the products by the concentration of the reactants at equilibrium. The magnitude of the chemical equilibrium constant can shed light on the composition of the mixture at equilibrium. Since the equilibrium constant given in the question is small, the equilibrium mixture would contain more reactants than products. If the equilibrium constant were larger than 1, this would indicate that the equilibrium mixture was made up of more products than reactants. Note: the chemical equilibrium constant is not an indicator of the rate of the reaction, only the concentrations of the equilibrium mixture.

How well did you know this?
1
Not at all
2
3
4
5
Perfectly
17
Q

The concentration of a reactant in a first-order decomposition reaction is decreased to 1/8 of its original value in 100 s. What is the half-life of the reactant?

A) 12.5 s
B) 25 s
C) 33.3 s
D) 50 s

A

C

You can test each of the answers: for 33.3 s intervals, the first 33.3 s finds the concentration dropping by 1/2, the next down to 1/4, the third down to 1/8, for a total of 100 s.

How well did you know this?
1
Not at all
2
3
4
5
Perfectly
18
Q

Galvanizing is a process used to protect steel by applying a sacrificial coating? Which metal is used in galvanizing?

A) Zinc
B) Gold
C) Copper
D) Aluminum

A

A

To provide a sacrificial coating the coating metal must be more reactive than the base metal being protected. Thus gold and copper are not applicable as they are both less reactive than iron and can only provide a passive coating.

Aluminum and zinc are more reactive than iron. Aluminum and zinc both form protective oxide layers that will stop corrosion of the iron, but aluminum is too reactive to be used in exposed weathering conditions. It is zinc that has just the right degree of reactivity that allows it to both protect the iron and not be corroded away, providing a strong and long lasting sacrificial coating.

How well did you know this?
1
Not at all
2
3
4
5
Perfectly
19
Q

Which of the following molecules are Lewis acids? (Me = methyl)

BF3, NH3, Me3C+ and H2O

A) BF3 and H2O
B) NH3 and Me3C+
C) BF3 and Me3C+
D) NH3 and H2O

A

C

A Lewis acid is an electron pair acceptor by virtue of it having an empty orbital. Both BH3 and CH3C+ are electron deficient, and therefore are Lewis acids. {Mnemonic: lEwis Acid = Electron Acceptor}

How well did you know this?
1
Not at all
2
3
4
5
Perfectly
20
Q

For the reaction 2NO2 –> N2O4, which of the following is true?

I. Adding NO2 shifts the equilibrium to the right.
II. Increasing the volume of the container shifts the equilibrium to the right.
III. Adding helium gas to the container shifts the equilibrium to the right.

A) I only
B) I and II only
C) I and III only
D) III only

A

A

The reaction shifts to the right to accommodate added NO2. Increasing the volume has the reverse effect – there’s more room for the more numerous NO2’s. Adding an inert gas raises the total pressure, but only partial pressures of reactants and products figure into the equilibrium expression.

How well did you know this?
1
Not at all
2
3
4
5
Perfectly
21
Q

Which of the following compounds has the greatest ionic character in its bonds? (You may consult a Periodic Table)

A) LiCl
B) CaCl2
C) BCl3
D) RbCl

A

D

Ionic character increases with increasing electronegativity of the potential anion and decreasing electronegativity of the potential cation. Electronegativity increases as one moves to the top right-hand corner of the periodic table, and decreases in the opposite direction.

Since the potential anion of all four compounds in question is chloride, we only need to find the compound with the cation located furthest toward the bottom left-hand corner of the periodic table. This compound is RbCl.

How well did you know this?
1
Not at all
2
3
4
5
Perfectly
22
Q

A cobalt electrode is immersed in 1.0M Co2+ and a lead electrode is immersed in 1.0 M Pb2+. Given are standard reduction potentials.

Co(2+) + 2e- -> Co ; E(reduction)= -0.28 V
Pb(2+) + 2e- -> Pb ; E(reduction)= -0.13 V

Which of the following describes the cell?

A) E(cell) = -0.15, galvanic
B) E(cell) = 0.15 V, galvanic
C) E(cell) = -0.41 V, electrolytic
D) E(cell) = 0.41 V, galvanic

A

B

The standard electrode potential E(cell) for a cell is found using the equation:
E(cell) = E(oxidation) + E(reduction)
In this case, lead will be reduced because it has a higher reduction potential than cobalt. This means that cobalt is being oxidized. Remember that on the MCAT, you will always be given standard reduction potentials, so if we need to find an oxidation potential, we simply switch to the negative reduction potential. Our equations should now look like this:
Co -> Co2+ + 2e- ; E(oxidation)= +0.28 V
Pb2+ + 2e- -> Pb ; E(reduction)= -0.13 V
Thus, using the first equation, we arrive at
E(cell) = (+0.28 V) + (-0.13 V) = 0.15 V
Remember that if E(cell) > 0, the reaction is spontaneous and happens in a galvanic cell. (The opposite is a non-spontaneous, electrolytic cell). Also, lead cobalt batteries were used in some of the first electric cars!

How well did you know this?
1
Not at all
2
3
4
5
Perfectly
23
Q

Which of the following best describes the process of oxidation?

A) The process in which electrons are added
B) The process in which oxygen is produced
C) The process in which hydrogen content increases
D) The process in which the oxidation number increases

A

D

Oxidation is the loss of electrons or an increase in oxidation state by a molecule, atom, or ion. The mnemonic LEO is A GERC stands for Loss Electrons Oxidation is Anode, Gain Electrons Reduction Cathode.

How well did you know this?
1
Not at all
2
3
4
5
Perfectly
24
Q

Which is a strong acid?

A) CH3CH2COOH
B) H2SO4
C) CH3CH2OH
D) NH4OH

A

B

A strong acid must be able to provide large numbers of hydrogen ions when dissolved in water, (H+(aq)). Thus a strong acid must dissociate (break into ions) completely upon dissolving in water to provide as many H+(aq) ions as possible.

All these compounds are water soluble, but only CH3CH2COOH and H2SO4 can provide H+(aq) ions. CH3CH2COOH or propanoic acid, is only partially dissociates (<1%) in water and is thus a weak acid. H2SO4 or sulfuric acid, completely (100%) dissociates into sulfate ions and hydrogen ions upon dissolving in water and is thus the strong acid of the group.

CH3CH2OH – ethanol does not dissociate, but stays as a complete molecule in solution.
NH4OH – ammonium hydroxide, a weak base which partially dissociates to give few hydroxide OH–(aq) ions.

How well did you know this?
1
Not at all
2
3
4
5
Perfectly
25
Q

Sodium hypochlorite (NaClO) is used as a disinfectant and has also been shown to treat chronic skin inflammation. What is the oxidation number of chlorine in NaClO?

A) -1
B) 0
C) +1
D) +2

A

C

In NaClO (sodium hypochlorite), sodium carries its typical +1 charge, oxygen carries its typical -2 charge. This means that the chlorine atom must carry a +1 charge in order to balance the overall charge of zero.

How well did you know this?
1
Not at all
2
3
4
5
Perfectly
26
Q

Which set of 1 Molar solutions has the correct pH values?

A) Water pH = 0; hydrochloric acid pH = 4; acetic acid pH = 7; ammonia solution pH = 10; sodium hydroxide solution pH = 14
B) Acetic acid pH = 0; hydrochloric acid pH = 4; water pH = 7; sodium hydroxide solution pH = 10; ammonia solution pH = 14
C) Sodium hydroxide solution pH = 0; ammonia solution pH = 4; water pH = 7; acetic acid pH = 10; hydrochloric acid pH = 14
D) Hydrochloric acid pH = 0; acetic acid pH = 4; water pH = 7; ammonia solution pH = 10; sodium hydroxide solution pH = 14

A

D

Acids have pH values of less than 7, with the stronger the acid the lower the pH. Bases have pH values of greater than 7, with the stronger the base the higher the pH. Typically:

Strong acids have a pH of 0 or less, hydrochloric acid pH = 0.
Weak acids between pH 3-6, acetic acid pH = 4.
Neutral solutions have pH = 7, water pH = 7.
Weak bases have a pH 8-10, ammonia solution pH = 10.
Strong bases have a pH of 14 or more, sodium hydroxide solution pH = 14.

How well did you know this?
1
Not at all
2
3
4
5
Perfectly
27
Q

A zero order reaction takes 5 minutes to decrease the original concentration of the reactants by half. How much longer will it take (in minutes) to have the concentration drop to a quarter of its original concentration?

A) 2.5
B) 5
C) 7.5
D) 10

A

A

Zero order reactions proceed at a constant rate. The time it takes to go from one-half to one-quarter of the original amount will be only half the time needed to go from all of the reactants to one-half of them. (Fun Fact: A first-order reaction would take 5 minutes.)

How well did you know this?
1
Not at all
2
3
4
5
Perfectly
28
Q

A reaction is found to have an equilibrium constant of 600 at a temperature of 300K and a pressure of 1.0 atm. It can be concluded that at these conditions:

I. ΔH < 0 II. ΔS < 0 III. ΔG < 0

A) I only
B) II only
C) III only
D) I and II only

A

C

Since K > 0, the reaction is spontaneous, and ΔG < 0; ΔG = -RTln(k) Although ΔH and ΔS combine to determine ΔG, we lack information to determine the sign of either of them.

How well did you know this?
1
Not at all
2
3
4
5
Perfectly
29
Q

The electron configuration of Cd2+ is best described by which of the following?

A) [Kr] 4s^25d^10
B) [Kr] 4d^10
C) [Kr] 5s^24d^10
D) [Kr] 5s^24d^8

A

B

Cadmium in it’s basic state has Configuration [Kr] 5s^24d^10. Positively charged cadmium (II) has lost 2 electrons. It will lose its two 5s-electrons first, making B the most reasonable answer. Answer A is illogical, as it has a higher shell number (5) for the d-electrons than the s-electrons (4). C is the correct configuration for neutral cadmium. Finally, D is incorrect because the d-electrons were removed rather than the s-electrons.

How well did you know this?
1
Not at all
2
3
4
5
Perfectly
30
Q

An electrolytic cell is used to reduce Ti(3+) to Ti metal. If a current of 3.5 A is passed through a solution of Ti(3+) for 2.0 min, which expression below can be used to calculate the moles of Ti deposited?

A) 3(2.0)(3.5)
B) (3.5)(2.0)(60)/(3)
C) (3.5)(2.0)(60)(3)/(96,500)
D) (3.5)(2.0)(60)/(96,500)(3)

A

D

Rearrange Faraday’s Law: moles = I x t/n x F. Note that we have to convert min to s, and that 3 electrons are transferred in the redox reaction.

How well did you know this?
1
Not at all
2
3
4
5
Perfectly
31
Q

During the reaction to form ammonia from nitrogen and hydrogen according to the reaction: N2(g) + 3 H2(g) <-> 2 NH3(g), ammonia was being produced at the rate of 1.50 mol.L-1.s-1.

What were the respective rates of consumption of nitrogen and hydrogen at this time?

A) N2(g) 1.50 and H2(g) 1.50 mol.L-1.s-1
B) N2(g) 2.25 and H2(g) 0.75 mol.L-1.s-1
C) N2(g) 0.75 and H2(g) 2.25 mol.L-1.s-1
D) N2(g) 3.00 and H2(g) 1.00 mol.L-1.s-1

A

C

The rate of consumption of the reactants will be the product of the rate of formation of the product multiplied by the ratio of the (co-efficient of the reactant) / (co-efficient of the product).

Rate N2(g) = 1.50 x 1/2 = 0.75 mole.L-1.s-1 and H2(g) 1.50 x 3/2 = 2.25 mole.L-1.s-1.

How well did you know this?
1
Not at all
2
3
4
5
Perfectly
32
Q

When a catalyst is added to a chemical reaction, what will be its effect on the energy of the activated complex and on the rate of the reaction?

A) The energy of the activated complex will increase, and the reaction rate will decrease
B) The energy of the activated complex will decrease, and the reaction rate will increase
C) The energy of the activated complex and the reaction rate will increase
D) The energy of the activated complex and the reaction rate will decrease

A

B

A catalyst lowers the activation energy of a reaction, causing an increase in the reaction rate.

How well did you know this?
1
Not at all
2
3
4
5
Perfectly
33
Q

A chemist runs a reaction that results in more stable bonds and orderly arrangement of the products versus the reactants. Which of the following is true of this reaction?

A) The enthalpy and entropy changes are positive
B) The enthalpy and entropy changes are negative
C) The enthalpy change is positive, and the entropy change is negative
D) The enthalpy change is negative, and the entropy change is positive

A

B

If the products have more stable bonds, that means they have less energy, and that energy must have been released, hence the change of enthalpy is negative. If the products are in a more orderly arrangement than the reactants, then the entropy decreased; thus the change in entropy is also negative, and the only answer left is B.

How well did you know this?
1
Not at all
2
3
4
5
Perfectly
34
Q

A student finds that BaCO3 is more soluble in HNO3 than in plain water. Which of the following is a likely explanation for this effect?

A) The NO3– ion attaches preferentially to the Ba2+
B) Ba2+ is a strong base and reacts completely with the H+ from the nitric acid, causing the barium salt to dissociate
C) Carbonate is a weak base and reacts with the H+ from the strong acid, causing the barium salt to dissociate
D) The HNO3– reduces the Ba2+ to barium metal

A

C

Carbonate is a weak base and reacts with the H+ from the strong acid, causing the barium salt to dissociate.
CO32- reacts with H+ to form HCO3–, causing the BaCO3 to dissociate more than it would in neutral water.

How well did you know this?
1
Not at all
2
3
4
5
Perfectly
35
Q

The many steps of glycolysis all exist in a state of dynamic equilibrium between reactant and product concentrations. Which of the following actions does NOT affect the equilibrium position of a reaction?

A) Adding or subtracting heat
B) Adding or removing a catalyst
C) Increasing or decreasing concentrations of reactants
D) Increasing or decreasing volumes of reactants

A

B

The equilibrium of a reaction can be changed by several factors. Adding or subtracting heat would shift the equilibrium based on the enthalpy change of the reaction. Increasing reactant concentrations would shift the equilibrium in the direction of the product and the opposite would occur if reactant concentrations were decreased. Changing the volume of a reactant would affect any reaction with gaseous reactants or products. While adding or removing a catalyst would change the reaction rate, it would not change where the equilibrium lies.

How well did you know this?
1
Not at all
2
3
4
5
Perfectly
36
Q

Which of the following correctly expresses both a correct statement and a correct explanation for that statement?

A) The solubility of Fe(OH)3 is pH-dependent owing to the half-cell potential of Fe(3+)/Fe(2+).
B) The solubility of Fe(OH)3 is pH-dependent owing to the presence of [OH-] in the solubility expression.
C) The solubility product of Fe(OH)3 is pH-dependent owing to the presence of [OH-] in the solubility expression.
D) The solubility product of Fe(OH)3 is not pH-dependent since [H+] does not appear in the solubility expression.

A

B

Note solubility, not solubility product, is hydroxide-dependent. D is wrong because [H+] and [OH-] are related.

How well did you know this?
1
Not at all
2
3
4
5
Perfectly
37
Q

Which of the following is closest to the pH of a mixture made up of 0.1 M each formic acid (whose Ka is 1.8 x 10-4) and potassium formate?

A) 0.1
B) 2
C) 4
D) 7

A

C

This solution is a buffer, for which:

[H+] = Ka [HFor]/[For–] = Ka

pH = -log (1.8 x 10^-4) = 3.7

How well did you know this?
1
Not at all
2
3
4
5
Perfectly
38
Q

Given that a parent and a daughter nucleus are isotopes of the same element, the ratio of alpha to beta decays which produced the daughter must be which of the following?

A) 1 to 1
B) 1 to 2
C) 2 to 1
D) 2 to 3

A

B

Alpha decay results in the loss of two protons, while ordinary beta decay results in the gain of one proton. Since isotopes of the same element have the same number of protons, the number of protons lost by alpha decay must equal the number gained by beta decay. Therefore, twice as many beta decays as alpha decays must occur, so the ratio of alpha to beta decays is 1:2.

How well did you know this?
1
Not at all
2
3
4
5
Perfectly
39
Q

An electron moving at constant velocity enters the area between two charged plates, as shown below. Which of the paths is most consistent with the electron’s trajectory after leaving the area between the charged plates?

+—————————- A (curved up)
-> B (Straight line up)
C (Straight line parallel)
-______________________ D (Straight line down)

(B and D are roughly 45 degrees respectively)

A) A
B) B
C) C
D) D

A

B

While in the area between the plates, the negatively charged electron is attracted to the positive plate and so it would bend towards the positive plate, or “upwards” in the diagram. But after leaving the plates, there is no more net force acting on the electron. Therefore, the electron would continue in motion in a straight line according to Newton’s first law (inertia).

How well did you know this?
1
Not at all
2
3
4
5
Perfectly
40
Q

A student wishes to find the solubility of PbCl2, whose K(sp) = 7 x 10^-29. Which equation should she solve to do so?

A) 7 x 10^-29 = (x)(x)
B) 7 x 10^-29 = (x)(2x)
C) 7 x 10^-29 = (x)(x)^2
D) 7 x 10^-29 = (x)(2x)^2

A

D

Note that [Cl^-] = 2x.

How well did you know this?
1
Not at all
2
3
4
5
Perfectly
41
Q

According to the Arrhenius equation (k = Ae^-E/RT), if a reaction occurs at room temperature, what would you expect to happen if the temperature was increased by a factor of 4?

A) The k value would increase by a factor of 4
B) The k value would not change
C) The k value would increase, but only slightly
D) The k value would decrease, but only slightly

A

C

The correct answer is C. The Arrhenius equation is given by k = Ae^-E/RT, where A is the pre-exponential factor, R is the gas constant, T is the temperature, and E is the activation energy. Thus, if the temperature were increased by a factor of 4, then the exponent would decrease by a factor of 4, since T is in the denominator. However, since it is e-x, the actual value won’t change much. For example, exp(-1) = 0.36, but exp(-1/4) = 0.77. Thus, although the value of k will increase, it will not increase by a factor of 4, only slightly.

How well did you know this?
1
Not at all
2
3
4
5
Perfectly
42
Q

At a given temperature T in kelvin, the relationship between the three thermodynamic quantities including the change in Gibbs free energy (ΔG), the change in enthalpy (ΔH) and the change in entropy (ΔS), can be expressed as follows:

ΔG = ΔH – TΔS

The sublimation of carbon dioxide occurs quickly at room temperature. What might be predicted for the three thermodynamic quantities for the reverse reaction?

A) Only ΔS would be positive
B) Only ΔS would be negative
C) Only ΔG would be positive
D) All 3 would be negative

A

C

“The sublimation of carbon dioxide occurs quickly at room temperature” means that it is spontaneous and so ΔG must be negative (by definition). Sublimation means:

Solid CO2 + heat -> vapor

Entropy (randomness) is clearly increasing (thus positive ΔS) because we are moving from a structured, ordered solid to randomly moving gas particles. Heat is required so it is endothermic meaning ΔH is positive. The question is asking about the reverse reaction so all 3 signs are reversed: ΔG is now positive; ΔH is now negative; ΔS is now negative.

Going deeper: Notice that a negative ΔS multiplied by a –T (see the Gibbs free energy equation) creates a positive term which overshadows the effect of the negative ΔH and thus ΔG is still positive. Also, please keep in mind that sometimes the real MCAT will provide the Gibbs free energy equation but sometimes they won’t.

How well did you know this?
1
Not at all
2
3
4
5
Perfectly
43
Q

A 13 gram sample of an unknown hydrocarbon is a gas at room temperature (25 °C) and atmospheric pressure. Its volume is 11.2 liters. Which of the following could the hydrocarbon be? Note: the ideal gas constant is 0.082 (Latm)/(Kmol)

A) CH
B) C2H4
C) C2H2
D) C3H3

A

C

In this question, we use the ideal gas law, PV = nRT. First of all, we calculate the number of mols in the volume of 11.2 L. n = PV/RT, or n = (111.2)/(0.082298). This gives us roughly n= 0.5 mol. You can bypass this by remembering that there are 22.4 L in a gaseous mol at STP. Thus, there is 0.5 mol of a substance that we know weighs 13g. The molar mass must be 26 g/mol then. With the knowledge that C = 12g/mol and H = 1g/mol (standard knowledge), we find that the hydrocarbon should be C2H2.

How well did you know this?
1
Not at all
2
3
4
5
Perfectly
44
Q

Which of the following should be observed with the addition of a catalyst to a chemical reaction?

A) The formation of a new product
B) The consumption of the catalyst
C) The activation energy is lowered
D) The decrease in enthalpy

A

C

A catalyst speeds up a chemical reaction by lowering the activation energy of a reaction. The catalyst is never consumed, nor does it alter the reactants or products. The total enthalpy of the reaction is also not changed from what it would have been without the catalyst. Only the rate at which the reaction occurs and the activation energy is affected. The only answer option that is correct is [C].

How well did you know this?
1
Not at all
2
3
4
5
Perfectly
45
Q

A chemist has flask containing a mixture of two gases at STP. The average velocity of a Gas 1 molecule is twice that of a Gas 2 molecule. Among the following, Gases 1 and 2 are most likely:

He = 4 g/mol
Ne = 20 g/mol
Ar = 40 g/mol
Kr = 80 g/mol

A) He and Ar
B) He and Kr
C) Ne and Ar
D) Ne and Kr

A

D

Two gases having the same temperature have the same average molecular kinetic energy. Therefore, (1/2)m1v1^2 = (1/2)m2v2^2, which gives m2/m1 = (v1/v2)^2. Since v1/v2 = 2, we have m2/m1 = 2^2 = 4. Among the available choices, the only gases with a mass ratio close to 4 to 1 are krypton (83.8 g/mol) and neon (20.2 g/mol).

How well did you know this?
1
Not at all
2
3
4
5
Perfectly
46
Q

What is a possible electronic structure for an excited Cu(I) ion?

A) [Ar] 4s1 3d10
B) [Ar] 3d10
C) [Ar] 4s2 3d9
D) [Ar] 3d9 4p1

A

D

The ground state for Cu is [Ar] 4s1 3d10. Transition metals always lose their valence s electrons before any d electrons, so the ground-state electron configuration for Cu+ [(I) means +1] is [Ar] 3d10. The only excited configuration for Cu+ must be [Ar] 3d9 4p1. An alternate strategy is to look at the number of electrons, as Cu+ has 28, and this helps eliminate A and C as they do not have 28 electrons.

How well did you know this?
1
Not at all
2
3
4
5
Perfectly
47
Q

The Ksp of Ag3PO4 is approximately 2 x 10^-18. Which of the following would increase the Ksp?

A) Add a solution of Na3PO4
B) Raise the pH
C) Lower the pH
D) None of the above

A

D

Only temperature will change the Ka, Kb, Kw, Keq, and Ksp. The Ksp will be constant and not change under the conditions provided in the answer choices.

How well did you know this?
1
Not at all
2
3
4
5
Perfectly
48
Q

Which of the following statements do not apply to ideal gas behavior?

I. Gas molecules move more rapidly at higher temperatures
II. It is difficult to compress gases beyond certain pressures, owing to intermolecular repulsions
III. Gases liquify as they are cooled beyond a certain point

A) I only
B) II only
C) I and III only
D) II and III only

A

D

Ideal gases do indeed move faster when heated due to the increase in average kinetic energy. However, we assume that there are no intermolecular forces between molecules, therefore they are infinitely compressible in theory. We also assume that an ideal gas will not change into a liquid no matter the conditions.

How well did you know this?
1
Not at all
2
3
4
5
Perfectly
49
Q

A liquid that has stronger cohesive forces than adhesive forces would have which type of meniscus?

A) Flat
B) Concave
C) Convex
D) Parabolic

A

C

The meniscus is the curved upper surface of liquid. This is formed because of unequal cohesion and adhesion attractions. For example, in water the adhesive forces are greater than the cohesive forces, therefore the water is able to “walk up the sides” of the surface. In the case of water, a U-shaped or concave meniscus is formed. Mercury, for example has stronger cohesive attractions between the mercury molecules than adhesive attractions between the mercury and the glass surface, therefore producing a convex meniscus, which looks like an inverted U.

How well did you know this?
1
Not at all
2
3
4
5
Perfectly
50
Q

How many liters of 0.3M HCl can be made from a 2L stock solution of 1.5M HCl?

A) 10L
B) 12L
C) 6L
D) 20L

A

A

```
M1V1 = M2V2
0.3)(V1) = (1.5)(2
V1 = 10 L
~~~

How well did you know this?
1
Not at all
2
3
4
5
Perfectly
51
Q

As you move from left to right across a Period in the Periodic Table, which trend of elementary properties does NOT occur?

A) Metallic properties decrease
B) Atomic radius increases
C) Electronegativity increases
D) First ionization energy increases

A

B

As you move from left to right across a Period, metallic properties decrease (metals > metalloids > non-metals), atomic radius decreases (the atoms get smaller), electronegativity (the ability to attract electrons in a compound) increases, first ionization energy (the energy required to remove the outermost electron from a neutral atom) increases and electron affinity (the energy released when an electron is bonded to an atom) increases.

How well did you know this?
1
Not at all
2
3
4
5
Perfectly
52
Q

At 25C, a one liter sample of pure water has a vapor pressure of 18.5 torr. If 10 g of NaCl are added to the sample, the vapor pressure of the water will:

A) Decrease
B) Increase
C) Remain unchanged
D) Be equal to he vapor pressure of the added NaCl

A

A

The addition of a solute to a liquid always lowers the vapor pressure of the liquid.

How well did you know this?
1
Not at all
2
3
4
5
Perfectly
53
Q

Which of the following anions is the strongest Bronsted base?

A) CH3COO-
B) NO3-
C) HSO4-
D) H2PO4-

A

A

Carboxylic acids are considered to be among the strongest organic acids but in comparison with all types of acids, carboxylic acids are very weak. By far, the protonated form of answer choice A (acetic acid is the IUPAC name; ethanoic acid is the systematic way to name it) would be the weakest of that group (after all, we are comparing it with nitric acid, sulfuric acid and phosphoric acid).

Logically, a strong acid would create a weak conjugate base, otherwise the acid would not give up the proton readily. Thus the reverse is true: a weak acid has a strong conjugate base.

How well did you know this?
1
Not at all
2
3
4
5
Perfectly
54
Q

Describe the reaction below.

PCl3(g) + Cl2(g) -> PCl5(g); Delta(Hf) = -87.9kJ/mol

A) Spontaneous at all temperatures
B) Nonspontaneous at all temperatures
C) ΔGrxn < 0 at only low temperatures
D) Spontaneous at only high temperatures

A

C

On the MCAT, numbers can be intimidating, but remember that if the answer choices have no numbers in them, we should think conceptually instead of empirically. Gibbs equation helps us determine if a reaction is spontaneous or non-spontaneous. It is given as:
ΔG = ΔH – TΔS

In this case, the enthalpy change ΔH is a negative number (exothermic). We know that the entropy change (ΔS) is also negative (the products are more ordered than the reactants) because the reactants are 2 moles of gas and the products are only 1 mole of gas (a synthesis of phosphorus pentachloride). Our equation now be read differently as:
ΔG = (negative number) – temperature * (negative number)

Temperature is in Kelvin and thus always a positive number, but we must consider two possibilities regarding temperature: (1) If this positive temperature is relatively low, the second part of the equation is a small positive number. Thus, if you add a small positive number to a negative number, it will probably still be negative (ΔG<0). (2) If the positive temperature is relatively high, the second part of the equation is a large positive number. If you add a large positive number to a negative number, it will probably become positive (ΔG>0).

In both these cases, ambiguity is important because we don’t know the temperature or ΔS. The only answer that appropriately describes the case we have here is C.

How well did you know this?
1
Not at all
2
3
4
5
Perfectly
55
Q

An amino acid in a medium with a pH value above its isoelectric point would have:

A) A neutral charge
B) A net positive charge
C) A net negative charge
D) Either a net positive or net negative charge depending on the pKa value

A

C

A pH above the isoelectric point means that the amino acid is in relatively basic conditions. A base will accept a proton (H+) from the amino acid, leaving the amino acid negatively charged.

How well did you know this?
1
Not at all
2
3
4
5
Perfectly
56
Q

According to the second law of thermodynamics, which of the following reactions would be favored?

A) 2H2O + O2 → 2H2O2
B) ADP + Pi → ATP
C) 2H2O2 → 2H2O + O2
D) 2Na + Cl2 → 2NaCl

A

C

The second law of thermodynamics states that nature tends to favor a state of disorder or entropy. In other words, chemical reactions that are exergonic (release energy) and result in more products than reactants are favored. The only answer option that has more products formed in comparison to the reactants is answer option C.

How well did you know this?
1
Not at all
2
3
4
5
Perfectly
57
Q

When a chemical bond is between a strongly positive and a strongly negative element, the bond is called what?

A) A covalent bond
B) An ionic bond
C) A hydrogen bond
D) A polar covalent bond

A

B

An ionic bond is one in which both parts of the bond are highly positive or negative so that they are attracted strongly to each other.

How well did you know this?
1
Not at all
2
3
4
5
Perfectly
58
Q

2 Ag2O -> 4 Ag + O2

A chemist runs the above reaction and produces one mole of Ag. How many grams of oxygen gas are produced?

A) 2 g
B) 4 g
C) 8 g
D) 16 g

A

C

From the balanced equation, we can see that 4 moles of Ag are produced for every 1 mole of O2. Therefore, if 1 mole of Ag is produced, then (1/4) mole of O2 is produced. The mass of (1/4) mole of O2 is (1/4)(2)(16 g/mol) = 8 grams.

How well did you know this?
1
Not at all
2
3
4
5
Perfectly
59
Q

Which opposite Phase Changes occur at the same temperature for a pure substance?

A) Melting and sublimation
B) Evaporation and boiling
C) Condensation and sublimation
D) Boiling and condensation

A

D

Boiling and Condensation

Melting: solid to liquid and Freezing: liquid to solid occur at the Melting/Freezing Point.

Boiling: liquid to gas and Condensation: gas to liquid occur at the Boiling/Condensation Point.

Both evaporation: liquid to gas at the surface of the liquid and sublimation: solid to gas occur at any temperature.

How well did you know this?
1
Not at all
2
3
4
5
Perfectly
60
Q

For which of the gas-phase reactions below would delta S be expected to be negative?

I. 2H2O –> 2H2 + O2
II. 2H –> H2
III. Cl2 + PCl3 –> PCl5

A)I only
B) II only
C) I and II only
D) II and III only

A

D

In each of II and II, the number of moles of gaseous products is less than that of gaseous reactants; thus “disorder” decreases.

How well did you know this?
1
Not at all
2
3
4
5
Perfectly
61
Q

Which of the following is the electron configuration of Ca2+?

A) [Ar]
B) [Ar] 4s2
C) [Ar] 4s2 3d2
D) [Ar] 4s2 4p2

A

A

A calcium atom normally contains 20 electrons, so if it acquires a charge of +2, it must have lost 2 electrons, leaving just 18. Only choice A has 18 electrons.

How well did you know this?
1
Not at all
2
3
4
5
Perfectly
62
Q

For a very weak acid, you would expect the pH to be:

A) Higher than the pKa
B) Equal to the pKa
C) Lower than the pKa
D) Equal to double the pKa

A

C

A weak acid will not dissociate much, meaning that A-/HA will be very low, and log(A-/HA) will be negative. Thus, a weak acid with a pKa of 10 will likely have a pH around 8 or 9.

How well did you know this?
1
Not at all
2
3
4
5
Perfectly
63
Q

Photons and other energy sources can excite electrons into higher energy levels, though these excitations are typically short-lived. X-ray imaging succeeds because different tissue types are differently able to absorb the energy of a photon in order to excite an electron. Suppose an electron falls from n = 4 to its ground state, n = 1. Which of the following effects is most likely?

A) A photon is absorbed
B) A photon is emitted
C) The electron moves into a p-orbital
D) The electron moves into a d-orbital

A

B

Because the electron is moving into the n = 1 shell, the only subshell available is the 1s subshell, which eliminates choices (C) and (D). There will be some energy change, however, as the electron must lose energy to return to the minimum-energy ground state. That will require emitting radiation in the form of a photon.

How well did you know this?
1
Not at all
2
3
4
5
Perfectly
64
Q

The density of an unknown gas under STP conditions is 3.17 g.L-1. What is the molecular mass of the gas?

A) 3.17/24.5 = 0.13
B) 3.17/22.4 = 0.14
C) 3.17 x 24.5 = 77.7
D) 3.17 x 22.4 = 71.0

A

D

STP is Standard Temperature and Pressure or 0oC and 1 atm pressure. At STP 1 mole of an ideal gas occupies 22.4 L. A good approximation of the molecular mass of a gas is obtained by multiplying its density in g.L-1 by 22.4 L to attain the mass of 1 mole of gas. Hence: 3.17 x 22.4 = 71.0

24.5 L is the molar volume of an ideal gas under LTP or Laboratory Temperature and Pressure, a balmy 25oC and 1 atm pressure.

How well did you know this?
1
Not at all
2
3
4
5
Perfectly
65
Q

Which of the following is NOT needed to calculate the endpoint of a titration?

A) Initial volume of titrant
B) Final volume of titrant
C) Concentration of the titrant
D) pH of the titrant

A

D

During a titration procedure, a buret is lowered into a clean Erlenmeyer flask. The initial volume of titrant is recorded and a stream of titrant is delivered while constantly mixing the sample solution. Once the pH indicator changes colors we have reached the endpoint. The final volume is recorded and used in conjunction with the initial volume to find the volume of titrant used, and along with the concentration of the titrant to find how many moles of titrant were used. Finally, we can use stoichiometry of the titration reaction to calculate the number of moles of reactant in our solution. The pH of the titrant is not needed.

How well did you know this?
1
Not at all
2
3
4
5
Perfectly
66
Q

The following statement is a description of which Gas Law? “As a gas is heated under constant pressure its volume will increase in proportion to the change in temperature.”

A) Dalton’s law of Partial Pressures
B) Boyle’s gas law
C) Charles’s gas law
D) The Ideal gas law

A

C

Charles’s Law states: V T
Dalton’s law of Partial Pressures states: Ptotal = P1 + P2 + P3 + …
Boyle’s gas law states: P 1/V
The Ideal gas law states: PV = nRT

How well did you know this?
1
Not at all
2
3
4
5
Perfectly
67
Q

The rate at which a chemical reaction occurs is dependent on many factors. Which of the following will increase the rate of reaction?

A) Decreasing the temperature
B) Adding a catalyst
C) Decreasing the concentration of a reactant in solution
D) Decreasing the partial pressure of a reactant gas

A

B

The rate at which a chemical reaction occurs is increased by increasing the temperature, adding a catalyst, increasing the concentration of a reactant in solution, increasing the partial pressure of a reactant gas and increasing the surface area of a reactant solid.

Catalysts increase the rate of reaction by providing an alternative way for the reaction to continue with a lower activation energy, thus making the reaction easy to occur under any given conditions.

How well did you know this?
1
Not at all
2
3
4
5
Perfectly
68
Q

People who have high blood pressure often go on low sodium diets, in which they substitute salt (NaCl) for another compound. Which of the following would be an appropriate supplementary salt?

A) MgCl2
B) KCl
C) BeCl2
D) SrCl2

A

B

The correct answer is B. The best substitute is one that is the closest to Na on the periodic table. Remember, families in the periodic table have similar chemical properties. Thus, anything in the second group is unacceptable, eliminating answers A, C, and D.

How well did you know this?
1
Not at all
2
3
4
5
Perfectly
69
Q

For the Henderson-Hasselbalch equation, under what conditions would you expect the pKa of the acid to equal the pH?

A) When the acid has not dissociated at all
B) When the amount of conjugate base is the same as the acid
C) When the acid is completely dissociated
D) When the acid is a weak acid

A

B

For the pH to equal the pKa, log(A-/HA) must be zero, which only occurs at log(1), meaning A-/HA = 1. Thus, the amounts of acid and conjugate base would be equal. This means that roughly half of the acid has dissociated.

How well did you know this?
1
Not at all
2
3
4
5
Perfectly
70
Q

Which of the following is true?

A) The conjugate base of a strong acid is a stronger base
B) As pKa increases, acidity decreases
C) The conjugate acid of a weak base is a very weak acid
D) As pKa increases, acidity increases

A

B

A very low acid dissociation constant Ka means that the acid is weak and thus very little product is made. Since pKa is -logKa, the negative sign means they are always opposites. Thus low Ka means high pKa.

How well did you know this?
1
Not at all
2
3
4
5
Perfectly
71
Q

The value of the water dissociation constant Kw varies with temperature. Its value is normally given as 1.00 x 10-14 mol2 dm-6 at room temperature but 1.00 x 10-13 mol2 dm-6 at 60 °C. What is the pH of pure water at 60 °C?

A) Equal to 7.0 thus the water is neutral
B) Less than 7.0 thus the water is acidic
C) Less that 7.0 but the water is basic
D) Less that 7.0 but the water is neutral

A

D

t is not necessary to calculate anything to solve this problem but we will go through the steps.

The dissociation of water (note carefully that the ratio of the products is 1:1; also keep in mind that this is the basis of the neutrality of pure water: acid units = base units):

H2O + H2O <-> H3O+ + OH–

The expression for the self-ionization of water or the water dissociation constant:

Kw = [H3O+][OH–] = [H+][OH–]

At room temperature, keeping in mind that the dissociation of the ions is 1:1, we get: 10-14 = [H+][OH–] = [H+]2

And so [H+] = 10-7 and then -log[H+] = 7 = pH

Doing the identical math but using Kw = 10-13 yields a pH of 6.5 BUT because the ratio of the ions is still 1:1 (if not, you would not have done the calculation) thus it must be neutral.

How well did you know this?
1
Not at all
2
3
4
5
Perfectly
72
Q

An aqueous solution of a metal nitrate (“X”NO3) forms a precipitate when magnesium sulfate (MgSO4) is added. However, a separate sample of the metal nitrate does not form any precipitate when mixed with hydrogen perchlorate (HClO4). What could this metal “X” be?

A) Sodium (Na)
B) Zinc (Zn)
C) Lead (Pb)
D) Sulfur (S)
E) Potassium (K)
A

C

First of all, we can immediately disregard answers A, B, and E. Nearly all compounds containing sodium, potassium, or zinc are soluble. Thus, we are left with answers C and D. Of the two, we realize that sulfur does not bind with sulfate. Sulfur sulfate does not exist. We recognize as well that lead compounds are rarely soluble, but lead perchlorate is. Thus, answer C is the correct answer.

How well did you know this?
1
Not at all
2
3
4
5
Perfectly
73
Q

A sealed vessel holds air and liquid water. Which of the following conditions would greatly increase the amount of air dissolved into the water?

A) Increasing the size of the vessel
B) Decreasing the temperature of the container
C) Adding argon gas to the vessel
D) Increasing the temperature of the container

A

C

The correct answer is C. Temperature change does increase the solvation ability of gas into liquid, but not significantly. However, we can easily recognize that a warm soda loses air much faster than a cold soda. Thus, it seems that answer B could be correct. However, in a sealed vessel, the factor that will increase the amount of air dissolved most immediately is the increase of pressure. The pressure is increased when another gas in forced in the system, like argon. The increased pressure will drive the equilibrium to force more air into the water.

How well did you know this?
1
Not at all
2
3
4
5
Perfectly
74
Q

Which of the following has the greatest ionic character in its bonds?

A) RbCl
B) LiCl
C) BCl3
D) CaCl2

A

D

Since Calcium is ionic and has attached to two chlorides ions which are anions, D is the correct answer. Boron does not form ions very well. A and B only have one ionic bond each.

How well did you know this?
1
Not at all
2
3
4
5
Perfectly
75
Q

A catalyst would change which of the following?

A) Enthalpy
B) Activation energy
C) Entropy
D) Change of Enthalpy

A

B

A catalyst lowers the activation energy, Ea, of a reaction.

How well did you know this?
1
Not at all
2
3
4
5
Perfectly
76
Q

Calculate the concentration of F- ions in a 2 M solution of hydrogen fluoride, HF. The Ka of HF is 6.8 x 10^-4.

A) 1.4 x 10^-3 M
B) 1.2 x 10^-2 M
C) 1.8 x 10^-2 M
D) 3.7 x 10^-2 M

A

D

The dissociation of HF is given by
HF -> (H+) + (F-)
Let [H+] = [F-] = x. Although the actual equilibrium concentration of HF is (2-x) M due to dissociation, we assume that x is very small and ignore the difference. That is, we take [HF] = 2 M at equilibrium. Since the equilibrium expression for this reaction is Ka = [H+][F-]/[HF], we have Ka = x^2/2, so

x^2 = 2Ka = 2(6.8 x 10^-4) = 13.6 x 10^-4, so x = 3.7 x 10^-2.

How well did you know this?
1
Not at all
2
3
4
5
Perfectly
77
Q

Which of the following are true regarding ideal gas assumptions?

I. Gas molecules have no volume.
II. Pressure is caused by collisions against the walls of the container.
III. All collisions between molecules are elastic.

A) I only
B) I and II only
C) I and III only
D) I, II, and III

A

C

According to ideal gas assumptions, all gas molecules are very far away from other gas molecules and their volumes are negligible. Should two gas molecules collide, their collision will be elastic, meaning no kinetic energy is lost (the average kinetic energy can only be changed by heating/cooling the gas). While it is true pressure is caused by the collision of gas molecules against the walls of the container, this is not an assumption for an ideal gas, but rather a general fact of nature. We typically want our ideal gases at low pressures too.

How well did you know this?
1
Not at all
2
3
4
5
Perfectly
78
Q

2.0 moles of H2O vapor at 0.20 atm and 298K are allowed to expand so that volume doubles. The temperature is unchanged. What is the new pressure (in atm)?

A) 0.05
B) 0.1
C) 0.2
D) 0.4

A

B

Use P1V1 = P2V2. Temperature is constant and can be eliminated from the equation. If the volume increases, we know the pressure must decrease. Doubling the volume should then halve the pressure.

How well did you know this?
1
Not at all
2
3
4
5
Perfectly
79
Q

Hydrogen gas generated in a reaction is collected over water at 25 degrees Celsius and is found to have a pressure of 746 torr. The vapor pressure of water at 25 degrees Celsius is 24 torr. Which expression can be used to find the pressure of H2, in atm?

A) 24/760
B) 746/760
C) (746 + 24)/760
D) (746 – 24)/760

A

D

You must subtract the pressure of water vapor from the total pressure, and change torr to atm. 1 atm = 760 torr = 760 mm Hg.

How well did you know this?
1
Not at all
2
3
4
5
Perfectly
80
Q

A sample of H2 gas occupies .20 L at 100 degrees Celsius and 760 mm of Hg pressure. To find the number of moles of gas in the sample, a student should calculate which of the following?

A) (0.082)(100)/(760)(.20)
B) (0.082)(100)/(1.00)(.20)
C) (1.00)(0.20)/(0.082)(100)
D) (1.00)(0.20)/(0.082)(373)

A

D

Use n = PV/RT. Remember R is a constant and that all the units must be converted into atm of pressure, liters of volume, and Kelvin from Celsius.

How well did you know this?
1
Not at all
2
3
4
5
Perfectly
81
Q

Which of the following statements about van der Waals equation are true?

I. At low densities, a real gas behaves like an ideal gas
II. The “b” term in the equation describes intermolecular forces
III. The product PV equals a constant for real and ideal gases alike

A) I only
B) II only
C) I and II only
D) I and III only

A

A

At low densities the correction terms to P and V become very small. Both correction terms, nb and an^2/V^2, depend on the number of gas molecules in a given volume, which is small at low densities. PV is not constant for the real gas as it is for an ideal gas.

How well did you know this?
1
Not at all
2
3
4
5
Perfectly
82
Q

Which of the following compounds will not react with acetyl chloride?

A) NH3
B) CH3NH2
C) (CH3)2NH
D) (CH3)4N+Cl-

A

D

Quaternary nitrogen atoms cannot react to form an amide because it already has four bonds.

How well did you know this?
1
Not at all
2
3
4
5
Perfectly
83
Q

Which of the following will NOT change the pressure in a closed vessel containing N2 gas?

I. Increasing the temperature
II. Replacing half the N2 molecules with helium atoms
III. Doubling the volume of the vessel

A) I only
B) II only
C) II and III only
D) I and II only

A

B

The pressure exerted by a gas does not depend on its molecular weight or the number of atoms in its molecules. Changes in T or V, however, will affect the pressure.

How well did you know this?
1
Not at all
2
3
4
5
Perfectly
84
Q

What is the maximum number of electrons in an atom that can have n = 3, l = 2 quantum numbers?

A) 5
B) 8
C) 10
D) 15

A

C

From +2 through -2, m can have 5 values. Each m value can have 2 values of m(s) associated with it, for a total of 10 states (i.e., the 10 3d electrons).

How well did you know this?
1
Not at all
2
3
4
5
Perfectly
85
Q

Magnetic properties are due to repulsion. Which of the following ground state ions is/are diamagnetic?
This question requires the use of a periodic table.

I. Fe2+
II. Zn2+
III. Cu1+
IV. Ni2+
V. V2+

A) I and IV only
B) I, IV, and V only
C) II only
D) II and III only

A

D

A diamagnetic ion will have no unpaired subshells in its atomic orbitals. A paramagnetic ion will have one unpaired electron in its atomic orbitals, leading it to be slightly more magnetic due to the unbalance. For example, Zn2+ will lose 2 electrons from the 4s2 while leaving the 3d10 orbital full, leaving no unpaired electrons. Copper will lose electrons from the 4s2 orbital to fill the 3d10 orbital, a more stable configuration and thus a diamagnetic compound.

How well did you know this?
1
Not at all
2
3
4
5
Perfectly
86
Q

For those atoms in the first column of the periodic table which form stable positive ions, the ion usually has a considerably smaller radius than the neutral atom. What explains this effect?

A) The outer electron(s) effectively screen the nuclear charge
B) The ion has a greater nuclear charge
C) The neutral atom has a greater nuclear charge
D) The neutral atom has an outer electron in an orbital of higher “n” than the ion

A

D

The electron in the neutral atom occupies an s orbital of higher n than the valence electrons of the positive ion. The former electron will stay, on average, farther from the nucleus than the inner electrons.

How well did you know this?
1
Not at all
2
3
4
5
Perfectly
87
Q

Which of the following has (have) buffer properties?

I. 50 mL of .1 M HCl
II. 100 mL of .1 M acetic acid
III. 50mL of .1 M NaOH

A) I only
B) II only
C) III only
D) The solution resulting when II and III are mixed

A

D

By mixing a weak acid with NaOH, we convert some of the acetic acid to acetate. The resulting mixture of the acid and its conjugate base is a buffer, and [H+] will be within an offer of magnitude of Ka.

How well did you know this?
1
Not at all
2
3
4
5
Perfectly
88
Q

According to scientists, Oparin and Haldane, which of the following gases were present in the primitive atmosphere?

A) CH4, NH3, H2, H2O
B) CO2, NH3, H2, H2O
C) N2, O2, H2, H2O
D) CH4, NH3, O2, H2O

A

A

In the primitive atmosphere, no O2 was present as most organisms used chemosynthesis to create organic compounds. The Oparin and Haldane hypothesis states that methane (CH4), ammonia (NH3), hydrogen (H2), and water (H2O) were the only gases present. It was after the Miller and Urey experiment that it was proven that these gases and lightning could have given rise to simple organic compounds that then later evolved into more complex forms of life.

How well did you know this?
1
Not at all
2
3
4
5
Perfectly
89
Q

Which of the following number sets is possible?

A) n = 2; l = 2; mt = 1; ms = +1/2
B) n = 2; l = 1; mt = -1; ms = +1/2
C) n = 2; l = 0; mt = -1; ms = -1/2
D) n = 2; l = 0; mt = 1; ms = -1/2

A

B

The azimuthal quantum number l cannot be higher than n - 1, ruling out A. The mt number, which describes the chemical’s magnetic properties, can only be an integer value between -l and l. It cannot be equal to +/-1 if l = 0.; this would imply that an s-subshell has three orbitals (-1, 0, and 1) when we know it can only have one. This rules out C and D.

How well did you know this?
1
Not at all
2
3
4
5
Perfectly
90
Q

What is the maximum number of electrons allowed in a single atomic energy level in terms of the principal quantum number n?

A) 2n
B) 2n + 2
C) 2n^2
D) 2l^2 + 2

A

C

For any value of n, there will be a maximum of 2n^2 electrons; that is, two per orbital. This can also be determined from the periodic table. There are only two elements (H and He) that have valence electrons in the n = 1 shell. Eight elements (Li to Ne) have valence electrons in the n = 2 shell. This is the only equation that matches this pattern.

How well did you know this?
1
Not at all
2
3
4
5
Perfectly
91
Q

Which of the following equations describes maximum number of electrons that can fill a subshell?

A) 2l + 2
B) 4l + 2
C) 2l^2
D) 2l^2 + 2

A

B

This formula describes the number of electrons in terms of the azimuthal quantum number l, which ranges from 0 to n - 1, with n being the principal quantum number.

Subshell Azimuthal (l) # of electrons
s 0 2
p 1 6
d 2 10
f 3 14

How well did you know this?
1
Not at all
2
3
4
5
Perfectly
92
Q

Which of the following atoms only has paired electrons in its ground state?

A) Sodium
B) Iron
C) Cobalt
D) Helium

A

D

The only answer choice without unpaired electrons in its ground state is helium. Recall from the chapter that a diamagnetic substance is identified by the lack of unpaired electrons in its shell. A substance without unpaired electrons, like helium, cannot be magnetized by an external magnetic field and is actually slightly repelled. Elements that come at the end of a block (Group IIA, the group containing Zn, and the noble gases, most notably) have only paired electrons.

How well did you know this?
1
Not at all
2
3
4
5
Perfectly
93
Q

Which of the following isotopes of carbon is LEAST likely to be found in nature?

A) 6C
B) 12C
C) 13C
D) 14C

A

A

Recall that the superscript refers to the mass number of an atom, which is equal to the number of protons plus the number of neutrons present in an element. Sometimes a text will list the atomic number, Z, as a subscript under the mass number, A. According to the periodic table, carbon contains six protons; therefore, its atomic number is 6. Isotopes all have the same number of protons, but differ in the number of neutrons. Almost all atoms with Z > 1 have at least one neutron. Carbon is most likely to have a mass number of 12, for six protons and six neutrons, as in choice B. Choices C and D are possible isotopes that would have more neutrons than 12C. The 6C isotope is unlikely. It would mean that there are 6 protons and 0 neutrons. This would result in a highly unstable isotope.

94
Q

Which of the following best explains the inability to measure position and momentum exactly and simultaneously according to the Heisenberg uncertainty principle?

A) Imprecision in the definition of the meter and kilogram
B) Limits on accuracy of existing scientific instruments
C) Error in one variable is increased by attempts to measure the other
D) Discrepancies between the masses of nuclei and of their component particles

A

C

The limitations placed by the Heisenberg uncertainty principle are caused by limitations inherent in the measuring process: is a particle is moving, it has momentum, but trying to measure that momentum necessarily creates uncertainty in the position. Even if we had an exact definition of the meter, or perfect measuring devices, we still wouldn’t be able to measure position and momentum simultaneously AND exactly.

95
Q

Which of the following electronic transitions would result in the greatest gain in energy for a single hydrogen electron?

A) An electron moves from n = 6 to n = 2
B) An electron moves from n = 2 to n = 6
C) An electron moves from n = 3 to n = 4
D) An electron moves from n = 4 to n = 3

A

B

For the electron to gain energy, it must absorb energy from photons to jump up to a higher energy level. There is a bigger jump between n = 2 and n = 6 than there is between n = 3 and n = 4.

96
Q

Suppose that an atom fills its orbitals as shown:

^v ^ ^ ^
3s 3p

Such an electron configuration most clearly illustrates which of the following laws of atomic physics?

A) Hund’s rule
B) Heisenberg uncertainty principle
C) Bohr model
D) Rutherford model

A

A

The MCAT covers the topics in this chapter qualitatively more often than quantitatively. It is critical to be able to distinguish the fundamental principles that determine electron organization, which are usually known by the names of the scientists who discovered or postulated them. The Heisenberg uncertainty principle refers to the inability to know the momentum and position of a single electron simultaneously. The Bohr model was an early attempt to describe the behavior of a single electron in a hydrogen atom. The Rutherford model described a dense, positively charged nucleus. The element shown here, phosphorus, is often used to demonstrate Hund’s rule because it contains a half-filled p subshell. Hund’s rule explains that electrons fill empty orbitals first before doubling up electrons in the same orbital.

97
Q

How many total electrons are in a 133Cs cation?

A) 54
B) 55
C) 78
D) 132

A

A

The quickest way to solve this problem is to use the periodic table and find out how many protons are in Cs atoms; there are 55. Neutral Cs atoms would also have 55 electrons. A stable Cs cation will have a single positive charge because it has one unpaired s-electron. This translates to one fewer electron than the number of protons or 54 electrons.

98
Q

The atomic weight of hydrogen is 1.008 amu. What is the percent composition of hydrogen by isotope, assuming that hydrogen’s only isotopes are 1H and 2D?

A) 92% H, 8%D
B) 99.2% H, 0.8% D
C) 99.92% H, .08% D
D) 99.992% H, .008% D

A

B

The easiest way to approach this problem is to set up a system of two algebraic equations, where H and D are the percentages of H (mass = 1 amu) and D (mass = 2 amu), respectively. Your setup should look like the following system:

H + D = 1 (percent H + percent D = 100%)
1 H + 2 D = 1.008 (atomic weight calculation)

Rearranging the first equation and substituting into the second yields (1 - D) +2D = 1.008, or D = 0.008. 0.008 is 0.8%, so there is 0.8% D.

99
Q

Consider the two sets of quantum numbers shown in the table, which describe two different electrons in the same atom.

n l mt ms
2 1 1 +1/2
3 1 -1 +1/2

Which of the following terms best describes these two electrons?

A) Parallel
B) Opposite
C) Antiparallel
D) Paired

A

A

The terms in the answer choices refer to the magnetic spin of the two electrons. The quantum number ms represents this property as a measure of an electron’s intrinsic spin. These electrons’ spins are parallel, in that their spins are aligned in the same direction (ms = +1/2 for both species).

100
Q

Lithium and sodium have similar chemical properties. For example, both can form ionic bonds with chloride. Which of the following best explains this similarity?

A) Both lithium and sodium are positively charged
B) Lithium and sodium are in the same group of the periodic table
C) Lithium and sodium are in the same period of the periodic table
D) Both lithium and sodium have low atomic weights

A

B

The periodic table is organized into periods (rows) and groups (columns). Groups are particularly significant because they represent sets of elements with the same valence electron configuration, which in turn will dictate many of the chemical properties of those elements. Although choice A is true, the fact that both ions are positively charged does not explain the similarity in chemical properties; most metals produce positively charged ion. Choice C is not true because lithium and sodium are in the same group, not period. Finally, although lithium and sodium do have relatively low atomic weights, so do several other elements that do not share the same properties.

101
Q

Carbon and silicon are the basis of biological life and synthetic computing, respectively. While these elements share many chemical properties, which of the following best describes a difference between the two elements?

A) Carbon has a smaller atomic radius than silicon
B) Silicon has a smaller atomic radius than carbon
C) Carbon has fewer valence electrons than silicon
D) Silicon has fewer valence electrons than carbon

A

A

As one moves from top to bottom in a group (column), extra electron shells accumulate, despite the fact that the valence configurations remain identical. These extra electron shells provide shielding between the positive nucleus and the outermost electrons, decreasing the electrostatic attraction and increasing atomic radius. Because carbon and silicon are in the same group, and silicon is farther down in the group, silicon will have a larger atomic radius because of its extra electron shell.

102
Q

What determines the length of an element’s atomic radius?

I. The number of valence electrons
II. The number of electron shells
III. The number of neutrons in the nucleus

A) I only
B) II only
C) I and II only
D) I, II, and III

A

C

Atomic radius is determined by multiple factors. Of the choices given, the number of valence electrons does have an impact on the atomic radius. As one moves across a period (row), protons and valence electrons are added, and the electrons are more strongly attracted to the central protons. This attraction tightens the atom, shrinking the atomic radius. The number of electron shells is also significant, as demonstrated by the trend when moving down a group (column). As more electron shells are added that separate the positively charged nucleus from the outermost electrons, the electrostatic forces are weakened, and the atomic radius increases. The number of neutrons is irrelevant because it does not impact these attractive forces.

103
Q

Ionization energy contributes to an atom’s chemical reactivity. Which of the following shows an accurate ranking of ionization energies from lowest to highest?

A) First ionization energy of Be < second ionization energy of Be < first ionization energy of Li
B) First ionization energy of Be < first ionization energy of Li < second ionization energy of Be
C) First ionization energy of Li < first ionization energy of Be < second ionization energy of Be
D) First ionization energy of Li < second ionization energy of Be < first ionization energy of Be

A

C

Ionization energy increases from left to right, so the first ionization energy of lithium is lower than that of beryllium. Second ionization energy is always larger than first ionization energy, so beryllium’s second ionization energy should be the highest value. This is because removing an additional electron from Be+ requires one to overcome a significantly larger electrostatic force.

104
Q

Antimony is used in some antiparasitic medications - specifically those targeting Leishmania donovani. What type of element is antimony?

A) Metal
B) Metalloid
C) Halogen
D) Nonmetal

A

B

Antimony (Sb) is on the right side of the periodic table, but not far right enough to be a nonmetal. It certainly does not lie far enough to the right to fall in Group VIIA (Group 17), which would classify it as a halogen. While sources have rarely classified antimony as metal, it is usually classified as a metalloid.

105
Q

The properties of atoms can be predicted, to some extent, by their location within the periodic table. Which property or properties increase in the direction of the arrows shown? (Arrows show up and to right on the periodic table)

I. Electronegativity
II. Atomic Radius
III. First ionization energy

A) I only
B) I and II only
C) I and III only
D) I, II, and III

A

C

Electronegativity describes how strong an attraction an element will have for electrons in a bond. A nucleus with a larger effective nuclear charge will have a higher electronegativity; Zeff increases toward the right side of a period. A stronger nuclear pull will also lead to increased first ionization energy, as the forces make it more difficult to remove an electron. The vertical arrow can be explained by the size of the atoms. As size decreases, the positive charge becomes more effective at attracting electrons in a chemical bond (higher electronegativity), and the energy required to remove an electron (ionization energy) increases.

106
Q

Metals are often used for making wires that conduct electricity. Which of the following properties of metals explains why?

A) Metals are malleable
B) Metals have low electronegativities
C) Metals have valence electrons that can move freely
D) Metals have high melting points

A

C

All four descriptions of metals are true, but the most significant property that contributes to the ability of metals to conduct electricity is the fact that they have valence electrons that can move freely. Malleability is the ability to shape a material with a hammer, which does not play a role in conducting electricity. The low electronegativity and high melting points of metals also do not play a major role in the conduction of electricity.

107
Q

Which of the following is an important property of the group of elements shaded in the periodic table below? (Group 2 is shaded)

A) These elements are the best electrical conductors in the periodic table
B) These elements form divalent cations
C) The second ionization energy for these elements is lower than the first ionization energy
D) The atomic radii of these elements decrease as one moves down the column

A

B

This Group represents the alkaline earth metals, which form divalent cations, or ions with a +2 charge. All of the elements in Group IIA have two electrons in their outermost s subshell. Because loss of these two electrons would leave a full octet as the outermost shell, becoming a divalent cation is a stable configuration for all of the alkaline earth metals. Although some of these elements might be great conductors, they are not as effective as the alkali metals. Choice C is also incorrect because, although forming a divalent cation is a stable configuration for the alkaline earth metals, the second ionization energy is still always higher than the first. Finally, Choice D is incorrect because atomic radii increase when moving down a group of elements because the number of electron shells increases.

108
Q

When dissolved in water, which of the following ions is most likely to form a complex ion with H2O?

A) Na+
B) Fe2+
C) Cl-
D) S2-

A

B

Iron is a transition metal. Transition metals can often form more than one ion. Iron, for example can be Fe2+ or Fe3+. The transition metals, in these various oxidation states, can often form hydration complexes with water. Part of the significance of these complexes is that, when a transition metal can form a complex, its solubility within the related solvent will increase. The other ions given might dissolve readily in water, but because none of them are transition metals, they will not likely form complexes.

109
Q

How many valence electrons are present in elements in the third period?

A) 2
B) 3
C) The number decreases as atomic number increases
D) The number increases as the atomic number increases

A

D

This question is simple if one recalls that periods prefer to the rows in the periodic table, while groups or families refer to the columns. Within the same period, an additional valence electron is added with each step toward the right side of the table.

110
Q

Which of the following elements has the highest electronegativity?

A) Mg
B) Cl
C) Zn
D) I

A

B

This question requires knowledge of the trends of electronegativity within the periodic table. Electronegativity increases as one moves from left to right for the same reasons that effective nuclear charge increases. Electronegativity decreases as one moves down the periodic table because there are more electron shells separating the nucleus from the outermost electrons. In this question, chlorine is the furthest toward the top-right corner of the periodic table.

111
Q

Which of the following atoms or ions has the largest effective nuclear charge?

A) Cl
B) Cl-
C) K
D) K+

A

D

The effective nuclear charge refers to the strength with which the protons in the nucleus can pull on electrons. This phenomenon helps to explain electron affinity, electronegativity, and ionization energy. In choice A, the nonionized chlorine atom, the nuclear charge is balanced by the surrounding electrons: 17 p+/17 e-. The chloride ion has a lower effective nuclear charge because there are more electrons than protons: 17 p+/18 e-. Next, elemental potassium, has the lowest effective nuclear charge because it contains additional inner shells that shield its valence electrons from the nucleus. Ionic potassium has a higher effective nuclear charge than any of the other options do because it has the same electron configuration as Cl- (and the same amount of shielding from inner shell electrons as neutral Cl) but contains two extra protons in its nucleus: 19 p+/18 e-.

112
Q

Why do halogens often form ionic bonds with alkaline earth metals?

A) The alkaline earth metals have much higher electron affinities than the halogens
B) By sharing electrons equally, the alkaline earth metals and halogens both form full octets
C) Within the same row, the halogens have smaller atomic radii than the alkaline earth metals
D) The halogens have much higher electron affinities than the alkaline earth metals

A

D

Ionic bonds are formed through unequal sharing of electrons. These bonds typically occur because the electron affinities of the two bonded atoms differ greatly. For example, the halogens have high electron affinities because adding a single electron to their valence shells would create full valence shells. In contrast, alkaline earth metals have very low electron affinities and are more likely to be electron donors because the loss of two electrons would leave them with full valence shells. Choice A states the opposite and is incorrect because the halogens have high electron affinity and the alkaline earth metals have low electron affinity. Choice B is incorrect because equal sharing of electrons is a classic description of covalent bonding, not ionic. Choice C is a true statement, but is not relevant to why ionic bonds form.

113
Q

Which is the highest-energy orbital of elements with valence electrons in the n = 3 shell?

A) s-orbital
B) p-orbital
C) d-orbital
D) f-orbital

A

C

When n = 3, l = 0, 1, or 2. The highest value for l in this case is 2, which corresponds to the d subshell. Although the 3d block appears to be part of the fourth period, it still has the principal quantum number n = 3. In general, the subshells within an energy shell increase in energy as follows: s < p < d < f (although there is no 3f subshell).

114
Q

What is the character of the bond in carbon monoxide?

A) Ionic
B) Polar covalent
C) Nonpolar covalent
D) Coordinate covalent

A

B

Carbon monoxide, CO, has a triple bond between carbon and oxygen, with the carbon and oxygen retaining one lone pair. In polar covalent bonds, the difference in electronegativity between the bonded atoms is great enough to cause electrons to move disproportionately toward the more electronegative atom but not great enough to transfer electrons completely. This is the case for CO. Oxygen is significantly more electronegative than carbon, so electrons will be disproportionately carried on the oxygen, leaving the carbon atom with a slight positive charge.

115
Q

Which of the following molecules contains the oxygen atom with the most negative formal charge?

A) H20
B) CO3^2-
C) O3
D) CH2O

A

B

To answer this question, one must understand the contribution of resonance structures to average formal charge. In Choice B there are three possible resonance structures. Each of the three oxygen atoms carries a formal charge of -1 in two out of the three structures. This averages to approximately -2/3 charge on each oxygen atom, which is more negative than in the other answer choices. Both water and formaldehyde (A and D) have no formal charge on the oxygen. Ozone, Choice C, has a -1/2 on two of the three oxygens and a +1 charge on the central oxygen.

116
Q

Which of the following correctly ranks the compounds below by ascending boiling point?

I. Acetone
II. KCl
III. Kr
IV. Isopropyl alcohol

A) I < II < IV < III
B) III < IV < I < II
C) II < IV < I < III
D) III < I < IV < II

A

D

The key to answering this question is to understand the types of intermolecular forces that exist in each of these molecules because larger intermolecular forces correspond to higher boiling points. Kr is a noble gas with a full octet, so the only intermolecular forces present are London dispersion forces, the weakest type of intermolecular forces. Acetone and isopropyl alcohol are both polar, so both have dipole-dipole interactions, which are stronger than dispersion forces. However, isopropyl alcohol can also form hydrogen bonds, increasing its boiling point. Finally, the strongest interactions are ionic bonds, which exist in potassium chloride.

117
Q

Both CO3^2- and ClF3 have three atoms bonded to a central atom. What is the best explanation for why CO3^2- has trigonal planar electronic geometry while ClF3 has trigonal bipyramidal electronic geometry?

A) CO3^2- has multiple resonance structures, while ClF3 does not
B) CO3^2- has a charge of -2, while ClF3 has no charge
C) ClF3 has lone pairs on its central atom, while CO3^2- has none
D) CO3^2- has lone pairs on its central atom, while ClF3 has none

A

C

The central carbon in carbonate has no lone pairs. It has three resonance structures, each of which involves a double bond between carbon and one of the three oxygens. Having made four bonds, carbon has no further orbitals for bonding or to carry lone pairs. This makes carbonate’s geometry trigonal planar. Alternatively, ClF3 also has three bonds; however, chloride still maintains two extra lone pairs. These lone pairs each inhabit one orbital, meaning that the central chloride must organize five items about itself: three bonds to fluorides and two lone pairs. The best configuration for maximizing the distance between all of these groups is trigonal bipyramidal. Choices A and B are true statements but do not account for the difference in geometry.

118
Q

Which of the following has the largest dipole moment?

A) HCN
B) H2O
C) CCl4
D) SO2

A

A

The best way to approach this problem is to draw the structures of each of these molecules, then consider the electronegativity of each bond as it might contribute to an overall dipole moment. HCN is the correct answer because of the large differences in electronegativity aligned in a linear fashion. There is a strong dipole moment in the direction of nitrogen, without any other moments cancelling it out. Water has two dipole moments, one from each hydrogen pointing in the direction of the oxygen. The molecule is bent, and the dipole moments partially cancel out. There is a molecular dipole, but it is not as strong as in HCN. Sulfur dioxide has a similar bent configuration, and its dipole will again be smaller than that of HCN. Further, oxygen and sulfur do not have as large a difference in electronegativity, so even the individual bond dipoles are smaller than those in the other molecules. CCl4 has tetrahedral geometry. Although each of the individual C-Cl bonds is highly polar, the orientation of these bonds causes the dipoles to cancel each other out fully, yielding no overall dipole moment.

119
Q

Despite the fact that both C2H2 and HCN contain triple bonds, the lengths of these triple bonds are not equal. Which of the following is the best explanation for this finding?

A) In C2H2, the bond is shorter because it is between atoms of the same element
B) The two molecules have different resonance structures
C) Carbon is more electronegative than hydrogen
D) Nitrogen is more electronegative than carbon

A

D

Bond lengths decrease as the bond order increases, and they also decrease with larger differences in electronegativity. In this case, because both C2H2 and HCN have triple bonds, we cannot compare the bond lengths based on bond order. We must then rely on other periodic trends. The bond length decreases when moving to the right along the periodic table’s rows because more electronegative atoms have shorter atomic radii. The nitrogen in HCN is likely to hold its electrons closer, or in a shorter radius, than the carbons in C2H2.

120
Q

Which of the following is the best explanation of the phenomenon of hydrogen bonding?

A) Hydrogen has a strong affinity for holding onto valence electrons
B) Hydrogen can only hold two valence electrons
C) Electronegative atoms disproportionately carry shared electron pairs when bonded to hydrogen
D) Hydrogen bonds have ionic character

A

C

Electronegative atoms bonded to hydrogen disproportionately pull covalently bonded electrons toward themselves, which leaves hydrogen with a partial positive character. That partial positive character is attracted to nearby negative or partial negative charges, such as those on other electronegative atoms.

121
Q

Which of the following best describes the number and character of the bonds in an ammonium cation?

A) Three polar covalent bonds
B) Four polar covalent bonds, of which none are coordinate covalent bonds
C) Four polar covalent bonds, of which one is a coordinate covalent bond
D) Four polar covalent bonds, of which two are coordinate covalent bonds

A

C

First recall that ammonium is NH4+ while ammonia is NH3. Ammonium is formed by the association of NH3, an uncharged molecule with a lone pair on the nitrogen, with a positively charged hydrogen cation. In other words, NH3 is a Lewis base, while H+ is a Lewis acid. This type of bonding between a Lewis acid and base is a coordinate covalent bond.

122
Q

Although the octet rule dictates much of molecular structure, some atoms can violate the octet rule by being surrounded by more than eight electrons. Which of the following is the best explanation for why some atoms can exceed the octet?

A) Atoms that exceed the octet already have eight electrons in their outermost electron shell
B) Atoms that exceed the octet only do so when bonding with transition metals
C) Atoms that exceed the octet can do so because they have d-orbitals in which extra electrons can reside
D) Some atoms can exceed the octet because they are highly electronegative

A

C

All atoms in the third period or greater have d-orbitals, which can hold an additional 10 electrons. The typical “octet” electrons reside in s and p-orbitals, but elements in period 3 or higher can place electrons into these d-orbitals.

123
Q

Which of the following types of intermolecular forces provides the most accurate explanation for why nobles gases can liquefy?

A) Hydrogen bonding
B) Ion-dipole interactions
C) Dispersion forces
D) Dipole-dipole interactions

A

C

All of the listed types of forces describe interactions between different types of molecules. However, noble gases are entirely uncharged and do not have polar covalent bonds, ionic bonds, or dipole moments. Therefore, the only intermolecular forces experienced by noble gases are London dispersion forces. Although these interactions are small in magnitude, they are necessary for condensation into a liquid.

124
Q

Which of the following is the best name for the new bond formed in the reaction?

H
\ ( H – O – H ) +
O + H+ = ( I )
/ ( H )
H

A) Nonpolar covalent bond
B) Ionic bond
C) Coordinate covalent bond
D) Hydrogen bond

A

C

The reaction in this question shows a water molecule, which has two lone pairs of electrons on the central oxygen, combining with a free hydrogen cation. The resulting molecule, H3O+ has formed a new bond between H+ and H2O. This bond is created via the sharing of one of oxygen’s lone pairs with the free H+ ion. This represents the donation of a shared pair of electrons from a Lewis base (H2O) to a Lewis acid (H+, electron acceptor). This type of bond is called a coordinate covalent bond.

125
Q

Both BF3 and NH3 have three atoms bonded to the central atom. Which of the following is the best explanation for why the geometry of these two molecules is different?

A) BF3 has three bonded atoms and no lone pair, which makes its geometry trigonal pyramidal
B) NH3 is polar, while BF3 is not
C) NH3 has three bonded atoms and one lone pair, which makes its geometry trigonal pyramidal
D) BF3 is nonpolar, while NH3 is polar

A

C

NH3 has three hydrogen atoms bonded to the central nitrogen, which also has a lone pair. These four groups (three atoms, one lone pair) lead NH3 to assume tetrahedral electronic geometry yet trigonal pyramidal molecular geometry. The nitrogen in ammonia is sp3-hybridized. By hybridizing all three p-orbitals and the one s-orbital, four groups are arranged about the central atom, maximizing distances between the groups to minimize the energy of the molecule with a tetrahedral configuration. In contrast, BF3 has three atoms and no lone pairs, resulting in sp2-hybridization. Its shape is called trigonal planar.

126
Q

Which of the following best describes an important property of bond energy?

A) Bond energy increases with increasing bond length
B) The more shared electron pairs comprising a bond, the higher the energy of that bond
C) Single bonds are more difficult to break than double bonds
D) Bond energy and bond length are unrelated

A

B

This answer requires an understanding of the trends that cause higher or lower bond energies. Bonds of high energy are those that are difficult to break. These bonds tend to have more shared pairs of electrons and, thus, cause a stronger attraction between the two atoms in the bonds. This stronger attraction also means that the bond length of a high-energy, high-order bond such as a triple bond is short than that of its lower-energy counterparts such as single or double bonds.

127
Q

Which of the following best describes ionic compounds?

A) Ionic compounds are formed from molecules containing two or more atoms
B) Ionic compounds are formed of charged particles and are measured by molecular weight
C) Ionic compounds are formed of charged particles that share electrons equally
D) Ionic compounds are three-dimensional arrays of charged particles

A

D

Ionic compounds are composed of atoms held together by ionic bonds. Ionic bonds associate charged particles with large differences in electronegativity. Rather than forming molecules or being measured by molecular weight, ionic compounds form large arrays of ions in crystalline solids and are measured with formula weights. In ionic bonds, electrons are not really shared but rather are donated from the less electronegative atom to the more electronegative atom.

128
Q

Which of the following compounds has a formula weight between 74 and 75 grams per mole?

A) KCl
B) C4H10O
C) MgCl2
D) BF3

A

A

Of the compounds listed, Choices B and D are covalent compounds and thus are measured in molecular weights, not formula weights. The formula weights of MgCl2 is much too high (24.3 amu / 2 x 35.5 amu = 95.3 amu per formula unit). Only KCl fits the criteria (39.1 amu + 35.5 amu = 74.6 amu).

129
Q

Which of the following is the gram equivalent weight of H2SO4 with respect to protons?

A) 49.1g
B) 98.1g
C) 147.1g
D) 196.2g

A

A

First, it is helpful to know the molar mass of one mole of H2SO4, which is found by adding the atomic weights of the atoms that constitute for molecule:

(2 x 1.0g/mol H) + (1 x 32.1g/ mol S) + (4 x 16.0g/mol O) = 98.1g/mol H2SO4

Gram equivalent weight is the mass (in grams) that would release one mole of protons. Because sulfuric acid has two hydrogens per molecule, the gram equivalent weight is 98.1g divided by 2, or 49.1g.

130
Q

In which of the following compounds is the percent composition of carbon by mass closest to 62%?

A) Acetone (C3H6O)
B) Ethanol (C2H5OH)
C) Propane (C3H8)
D) Methanol (CH3OH)

A

A

The percent composition by mass of any given element within a molecule is equal to the mass of that element in the molecule divided by the molar mass of the compound, times 100%. In this case:

(C = 6) (H = 1) (O = 16)

C3H6O
(3x12g/mol) / (58g/mol C3H6O) = 18/29 = 2/3 = 66.7% C

C2H5OH
(2x12g/mol) / (46g/mol C2H5OH) = 50% C

C3H8
(3x12g/mol) / (44g/mol C3H8) = 9/11 = 80% C

CH3OH
(12g/mol) / (32g/mol) = 3/8 = 37.5% C

Note that all four of these compounds are commonly encountered on the MCAT, and you should be familiar with the structure and composition of each, including their common names.

131
Q

What is the specific characterization of the reaction above?

Ca(OH)2(aq) + H2SO4(aq) -> CaSO4(aq) + H2O(l)

A) Single-displacement
B) Neutralization
C) Double-displacement
D) Oxidation-reduction

A

B

This reaction is a classic example of a neutralization reaction, in which an acid and a base react to form a salt and, usually, water. Although this reaction also fits the criteria for a double-displacement reaction in which two molecules essentially exchange ions with each other, neutralization is a more specific description of the process.

132
Q

In the reaction shown, if 39.05g of Na2S are reacted with 85.5g of AgNO3, how much of the excess reagent will be left over once the reaction has gone to completion?

Na2S + 2 AgNO3 -> Ag2S + 2 NaNO3

{Na = 23g] [S = 32.1g] [Ag = 107.9] [N = 14] [O = 16]

A) 19.5g Na2S
B) 26.0g Na2S
C) 41.4g AgNO3
D) 74.3g AgNO3

A

A

In this question, you are first given the masses of both reactant used to start the reaction. To figure out what will be left over, we must first determine which species is the limiting reagent.

The formula weight of Na2S:

(2x23g/mol Na) + (1x32.1g/mol S) = 78.1 g/mol Na2S.

The formula weight of AgNO3:

(1x107.9g/mol Ag) + (1x14g/mol N) + (3x16g/mol O) = 169.9g/mol AgNO3.

From this we can determine that we are given:

  1. 05g Na2S / 78.1g/mol = 0.5 mol Na2S
  2. 5g AgNO3 / 169.9g/mol = 0.5 mol AgNO3

Because we need two moles of AgNO3 for every mole of Na2S, AgNO3 is the limiting reagent, and the correct answer choice will be given in grams of Na2S. If 0.5 mol of AgNO3 are used up, and Na2S will be consumed at half the rate of AgNO3 (based on their mole ratio), then 0.25 mol Na2S will be used up. We then have 0.25 mol excess Na2S, which has a mass of:

0.25 mol x 78.1g/mol = 19.5g Na2S.

133
Q

Aluminum metal can be used to remove tarnish from silver when the two solid metals are placed in water, according to the following reaction:

3 AgO + 2 Al -> 3 Ag + Al2O3

This reaction is a:

I. Double-displacement reaction
II. Single-displacement reaction
III. Oxidation-reduction reaction
IV. Combination reaction

A) II only
B) IV only
C) I and III only
D) II and III only

A

D

In the reaction, there is a single-displacement, with the silver in silver oxide being replaced by the aluminum to form aluminum oxide. This single-displacement reaction also necessitates a transfer of electrons in an oxidation-reduction reaction; silver, for example, changes from the +2 oxidation state to neutral. Aluminum changes from neutral to the +3 oxidation state.

134
Q

Which of the following types of reactions generally have the same number of reactant and products?

I. Double displacement reactions
II. Single-displacement reactions
III. Combination reactions

A) I only
B) II only
C) I and II only
D) II and III only

A

C

Typically, both single-displacement and double-displacement reactions have two reactants that swap either one or two components between the two species. Combination reactions, on the other hand, have more reactants than products because the reactants combine together to form the product.

135
Q

A reaction that utilizes oxygen and hydrocarbons as reactants and that produces carbon dioxide and water as products is best characterized as:

A) Single-displacement
B) Combustion
C) Metathesis
D) Decomposition

A

B

This description characterizes a combustion reaction because a hydrocarbon acts as a fuel when reacting with oxygen. Carbon dioxide (an oxide) and water are the products of such a reaction.

136
Q

In the process of photosynthesis, carbon dioxide and water combine with energy to form glucose and oxygen, according to the following equation:

CO2 + H2O -(hv)-> C6H12O6 + O2

What is the theoretical yield of glucose if 30g of water are reacted with excess carbon dioxide and energy, according to the equation above?

A) 30.0g
B) 50.0g
C) 300.1g
D) 1801g

A

B

The equation given is unbalanced, so the first step must be to balance it:

6 CO2 + 6 H2O -(hv)-> C6H12O6 + 6O2

The theoretical yield is the amount of product synthesized if the limiting reagent is completely used up. This question therefore asks how much glucose is produced if the limiting reagent is 30 grams of water. Using the three-fraction method to solve for the mass of glucose produced gives:

30.0g H2O x (1 mol H2O / 18g H2O) x
(1 mol C6H12O6 / 6 mol H2O) x
(180g C6H12O6 / 1 mol C6H12O6)
=
50g C6H12O6
137
Q

In the following reaction:

Au2S3 (s) + H2 (g) -> Au (s) + H2S (g)

If 2 moles of Au2S3 (s) is reacted with 5 moles of hydrogen gas, what is the limiting reagent?

A) Au2S3 (s)
B) H2 (g)
C) Au (s)
D) H2S (g)

A

B

A limiting reagent is by definition a reactant. Because Au and H2S are products, they cannot act as limiting reagents. Next, note that the given equation is unbalanced and the first step is to balance it:

Au2S3 (s) + 3 H2 (g) -> 2 Au (s) + 3 H2S (g)

The problem states that 2 moles of gold(III) sulfide and 5 moles of hydrogen has are available. To use up both moles of gold(III) sulfide, 6 moles of hydrogen gas are needed because there is a 1:3 ratio between these reactants. Since only 5 moles of hydrogen gas are present, that will have to be the limiting reagent.

138
Q

Which of the following would make the strongest electrolytic solution?

A) A nonpolar covalent compound with significant solubility
B) An ionic compound composed of one cation with a +3 charge and three anions with -1 charge
C) A polar covalent compound with a small dissociation constant
D) An ionic compound composed of two cations with +1 charge and one anion with -2 charge

A

B

The best electrolytes dissociate readily (have a high dissociation constant) and are ionic compounds with large amounts of cations and anions. Choice D has fewer total ions with a smaller total magnitude of charge and therefore is not as strong an electrolyte as in choice B.

139
Q

What is the molecular formula of a compound with an empirical formula of B2H5 and a molar mass of 53.2g/mol?

A) B2H5
B) B3H7
C) B4H10
D) B6H15

A

C

The simplest approach is to determine the molar mass of the empirical formula. B2H5 has a molar mass of 26.6g/mol. A molecular formula is ALWAYS a multiple of the empirical formula; doubling this quantity will result in the molar mass given in the question stem. Therefore, the compound must be B4H10.

140
Q

In a third-order reaction involving two reactants and two products, doubling the concentration of the first reactant causes the rate to increase by a factor of 2. What will happen to the rate of this reaction if the concentration of the second reactant is cut in half?

A) It will increase by a factor of 2
B) It will increase by a factor of 4
C) It will decrease by a factor of 2
D) It will decrease by a factor of 4

A

D

Based on the information given in the question, the rate is first-order with respect to the concentration of the first reactant; when the concentration of that reactant doubles, the rate also doubles. Because the reaction is third-order, the sum of the exponents in the rate law must be equal to 3. Therefore, the reaction order with respect to the other reactant must be 3 - 1 = 2. If the concentration of this second reactant is multiplied by 1/2, the rate will be multiplied by (1/2)^2 = 1/4.

141
Q

In a certain equilibrium process, the activation energy of the forward reaction (ΔGforward) is greater than the activation energy of the reverse reaction (ΔGreverse). This reaction is:

A) Endothermic
B) Exothermic
C) Spontaneous
D) Nonspontaneous

A

D

If the activation energy of the forward reaction is greater than the activation energy of the reverse reaction, then the products must have a higher free energy than the reactants. The overall energy of the system is higher at the end than it was in the beginning. The net free energy change is positive, indicating an endergonic (nonspontaneous) reaction. The terms endothermic and exothermic are associated with enthalpy (ΔH). While free energy does depend on enthalpy, it also depends on entropy (ΔS); there is not enough information in the question stem to reliably determine the sign of the entropy change of the reaction.

142
Q

A reactant in a second-order reaction at a certain temperature is increased by a factor of 4. By how much is the rate of the reaction altered?

A) It is unchanged
B) It is increased by a factor of 4
C) It is increased by a factor of 16
D) It cannot be determined from the information given

A

D

A second-order reaction can be second-order with respect to one reactant, or first-order with respect to two different reactants. In this case, one reactant was increased by a factor of 4. If the reaction is second-order with respect to this reactant, the rate law will be rate = k[A]^2[B]^0 and the rate will increase by a factor of 16. If it is first-order with respect to this reactant and first-order with respect to another reactant, the rate will be rate = k[A]^1[B]^1, and the rate will increase by a factor of 4. We do not know which of these is the correct rate law and, thus, cannot determine the effect on the rate.

143
Q

The concentrations of all reactants in a zero-order reaction are increased two-fold. What is the new rate of the reaction?

A) It is unchanged
B) It is decreased by a factor of 2
C) It is increased by a factor of 2
D) It cannot be determined from the information given

A

A

By definition, zero-order reactions are unaffected by the concentrations of any reactants in the reaction. Thus, changing the concentrations of these reactants will not affect the rate.

144
Q

Which of the following experimental methods should NEVER affect the rate of a reaction?

A) Placing an exothermic reaction in an ice bath
B) Increasing the pressure of a reactant in a closed container
C) Putting the reactants into an aqueous solution
D) Removing the product of an irreversible reaction

A

D

The question asks which alteration does NOT affect the rate of the reaction. Temperature directly affects the rate constant (k). Changing the partial pressure of a gas will affect the number of effective collisions per time. This will make Choice B incorrect but do note that concentration changes will not affect the rate of zero-order reactions. Solvents affect the rate of reactions depending on how the reactants interact with the solvent. Removing the product of an irreversible reaction should not affect the rate of the reaction because the rate law does not depend on the concentration of products.

145
Q

What would increasing the concentration of reactants accomplish in a solution containing a saturated catalyst?

A) It would increase the rate constant but not the reaction rate
B) It would decrease the rate constant but increase the reaction rate
C) It would increase the rate constant and increase the reaction rate
D) The reaction rate would be unaffected

A

D

While increasing the concentration of reactants can alter the reaction rate in first or higher-order reactions, saturated solutions containing a catalyst have a maximum turnover rate and cannot increase the rate constant or the reaction rate any higher by adding more reactant molecules.

146
Q

A certain chemical reaction has the following law:

rate = k[NO2][Br2]

Which of the following statements necessarily describe(s) the kinetics of this reaction?

I. The reaction is second-order
II. The amount of NO2 consumed is equal to the amount of Br2 consumed
III. The rate will not be affected by the addition of a compound other than NO2 and Br2

A) I only
B) I and II only
C) II and III only
D) I, II, and III

A

A

If the sum of the exponents (orders) of the concentrations of each species in the rate law is equal to 2, then the reaction is second-order. The exponents in the rate law are unrelated to stoichiometric coefficients, so NO2 and Br2 could have any stoichiometric coefficients in the original reaction and still be a second-order reaction, invalidating statement II. Statement III is incorrect because the rate can be affected by a wide variety of compounds. A catalyst, for example, could increase the rate.

147
Q

Which of the following best describes the purpose of a catalyst?

A) Catalysts are used up in the reaction, increasing reaction efficiency
B) Catalysts increase the rate of the reaction by lowering the activation energy
C) Catalysts alter the thermodynamics of the reaction to facilitate the formation of products or reactants
D) Catalysts stabilize the transition state by bringing it to a higher energy

A

B

By definition, a catalyst increases the rate of a reaction by lowering the activation energy, making it easier for both the forward and reverse reactions to overcome this energy barrier. Catalysts are neither used up in the reaction, nor do they alter the equilibrium of a reaction. Finally, catalysts stabilize the transition state by lowering its energy, not raising it.

148
Q

If the rate law for a reaction is:

rate = k[A]^0 [B]^2 [C]^1

What is the overall order for the reaction?

A) 0
B) 2
C) 3
D) 4

A

C

The overall order of a reaction is the sum of the individual orders in the reaction. Therefore, the rate order is 0 + 2 + 1 = 3.

149
Q

The following system obeys second-order kinetics.

2 NO2 -> NO3 + NO (slow)
NO3 + CO -> NO2 + CO2 (fast)

What is the rate law for this reaction?

A) rate = k[NO2] [CO]
B) rate = k[NO2]^2 [CO]
C) rate = k[NO2] [NO3]
D) rate = k[NO2]^2

A

D

To answer this question, recall that the slow step of a reaction is the rate-determining step. The rate is ALWAYS related to the concentrations of the reactants in the rate-determining step (NOT the overall reaction), so NO2 is the only compound that should be included in the correct answer. The concentration of NO2 is squared in the rate law because the question stem tells us that the system obeys second-order kinetics.

150
Q

A reaction is found to stop just before all reactants are converted to products. Which of the following could be true about this reaction?

A) The reaction is irreversible, and the forward rate is greater than the reverse rate
B) The reaction is irreversible, and the reverse rate is too large for products to form
C) The reaction is reversible, and the forward rate is equal to the reverse rate
D) The reaction is reversible, and the reverse rate is greater than the forward rate

A

C

This scenario likely describes a situation in which a reaction has reached equilibrium very far to the right (with high product concentration and low reactant concentration). This reaction must be reversible because the reaction did not proceed all the way to the right. Any reaction in equilibrium has equal forward and reverse rates of reaction.

151
Q

Carbonated beverages are produced by dissolving carbon dioxide in water to produce carbonic acid:

CO2 (g) + H2O (l) <-> H2CO3 (aq)

When a bottle containing carbonated water is opened, the taste of the beverage gradually changes as the carbonation is lost. Which of the following statements best explains this phenomenon?

A) The change in pressure and volume causes the reaction to shift to the left, thereby decreasing the amount of aqueous carbonic acid
B) The change in pressure and volume causes the reaction to shift to the right, thereby decreasing the amount of gaseous carbon dioxide
C) Carbonic acid reacts with environmental oxygen and nitrogen
D) Carbon dioxide reacts with environmental oxygen and nitrogen

A

A

Carbon dioxide gas evolves and leaves the bottle, which decreases the total pressure of the reactants. Le Châtelier’s principle explains that a decrease in pressure shifts the equilibrium to increase the number of moles of gas present. This particular reaction will shift to the left, which in turn will decrease the amount of carbonic acid and increase the amount of carbon dioxide and water. Oxygen and nitrogen are not highly reactive and are unlikely to combine spontaneously with carbon dioxide or carbonic acid.

152
Q

If Kc&raquo_space; 1:

A) The equilibrium mixture will favor products over reactants
B) The equilibrium mixture will favor reactants over products
C) The equilibrium concentrations of reactants and products are equal
D) The reaction is essentially irreversible

A

A

The larger the value of Keq (whether Kc or Kp), the larger the ratio of products to reactants. Therefore, if Kc&raquo_space; 1, there are significantly larger concentrations of products than reactants at equilibrium. Even with a large Keq, the reaction will ultimately reach equilibrium far toward the products side and is therefore reversible.

153
Q

Acetic acid dissociates in solution according to the following equation:

CH3COOH <-> CH3COO- + [H+]

If sodium acetate is added to a solution of acetic acid in excess water, which of the following effects would be observed in the solution?

A) Decreased pH
B) Increased pH
C) Decreased pKeq (pKa)
D) Increased pKeq (pKa)

A

B

Adding sodium acetate increases the number of acetate ions present. According to Le Châtelier’s principle, this change will push this reaction to the left, resulting in a decrease in the number of free H+ ions. Because pH is determined by the hydrogen ion concentration, a decrease in the number of free protons will increase the pH. An acid’s Ka (which is simply the Keq for acid dissociation) will remain constant under a given temperature and pressure.

154
Q

FeI (aq) + I2 (g) -> FeI3 (aq)

Which of the following would increase the formation of product?

A) Decreasing the volume of the container
B) Decreasing the pressure of the container
C) Increasing the volume of the container
D) Decreasing the volume of the container while maintaining a constant pressure

A

A

Both increasing the pressure of the container and decreasing the volume would favor the side with fewer moles of gas, which is the product side. The significance of decreasing the volume of the container in most equilibria is that there is an increase in pressure; in this case, however the pressure remains constant despite the change in volume (choice D).

155
Q

FeI (aq) + I2 (g) -> FeI3 (aq)

If this reaction were exothermic, what effect would decreasing the temperature have on the equilibrium?

A) The forward reaction rate and the reverse reaction rate both increase
B) The forward reaction rate decreases while the reverse reaction rate increases
C) The forward reaction rate increases while the reverse reaction rate decreases
D) The forward reaction rate and the reverse reaction rate both decrease

A

C

An exothermic reaction produces heat. Decreasing the temperature favors product formation, resulting in an increase in the forward reaction rate with a concomitant decrease in the reverse reaction rate.

156
Q

Which of the following actions does NOT affect the equilibrium position of a reaction?

A) Adding or removing heat
B) Adding or removing a catalyst
C) Increasing or decreasing concentrations of reactants
D) Increasing or decreasing volumes of reactants

A

B

The equilibrium of a reaction can be changed by several factors. Adding or subtracting heat would shift the equilibrium based on the enthalpy change (ΔH) of the reaction. Increasing reactant concentrations would shift the equilibrium in the direction of the product, and the opposite would occur if reactant concentrations were decreased. Changing the volume of a reactant would affect any reaction with gaseous reactants or products. While adding or removing a catalyst would change the reaction rates, it would not change where the equilibrium lies.

157
Q

Increasing temperature can alter the Keq of a reaction. Why might increasing temperature indefinitely be unfavorable for changing reaction conditions?

A) The equilibrium constant has a definite limit that cannot be surpassed
B) The products or reactants can decompose at high temperatures
C) Increasing temperature would decrease pressure, which may or may not alter reaction conditions
D) If a reaction is irreversible, its Keq will resist changes in temperature

A

B

At extremely high temperatures, reactants or products may decompose, which will affect the equilibrium and potentially destroy the desired products. Choice A implies that reactions have limits, which is true; however, this does not make increasing temperature unfavorable. Choice C is false because increasing temperature would also increase pressure, assuming constant volume. Choice D is incorrect because it refers to properties of irreversible reactions, which would not be involved in an equilibrium between products and reactants.

158
Q

Which of the following is true of equilibrium reactions?

I. An increase in k1 results in a decrease in k-1
II. As the concentration of products increases, the concentration of reactants decreases
III. The equilibrium constant is altered by changes in temperature

A) I only
B) II and III only
C) I and III only
D) I, II, and III

A

B

Statement I is false because the addition of a catalyst could increase the rate constants of both the forward and reverse reactions. Statement II is true because in order for products to come into existence, reactants must be used up. Statement III is also true; all K values are temperature-dependent.

159
Q

Compound A has a Ka (equilibrium constant of acid dissociation) of approximately 10^-4. Which of the following compounds is most likely to react with a solution of compound A?

A) HNO3
B) NO2
C) NH3
D) N2O5

A

C

Ka is equal to the ratio of products to reactants, with each species raised to its stoichiometric coefficient. A compound with a Ka greater than 10^-7 contains more [H+] cations than {HA-] anions at equilibrium, which makes it an acid. This means that the compound in question is likely to react with a compound that is basic. Of the four answer choices, NH3 is the only base.

160
Q

Consider the following two reactions:

3A + 2B <-> 3C + 4D (Reaction 1)
4D + 3C <-> 3A + 2B (Reaction 2)

If Keq for reaction 1 is equal to 0.1, what is Keq for reaction 2?

A) 0.1
B) 1
C) 10
D) 100

A

C

Reaction 2 is the reverse of reaction 1. This means that Keq for reaction 2 is the inverse of Keq of reaction 1, so the answer is 1/0.1 = [ 1 / (1/10) ] = 10.

161
Q

Which of the following statements best describes the effect of lowering the temperature of the following reaction?

A + B <-> C + D ΔH = -1.12kJ/mol

A) [C] and [D] would increase
B) [A] and [B] would increase
C) ΔH would increase
D) ΔH would decrease

A

A

A negative ΔH value indicates an exothermic reaction, meaning that the forward reaction produces heat. Visualize this as follows:

A + B <-> C + D + heat

This means that removing heat by decreasing the temperature is similar to removing any other product of the reaction. To compensate for this loss, the reaction will shift to the right, causing an increase in concentration of C and D, as well as a decrease in the concentrations of A and B.

162
Q

Consider the cooling of an ideal gas in a closed system. This process is illustrated in the pressure-volume graph shown in the following figure.

Hyperbolic - decreasing in P as V moves to the right (steep slope and slowly evens out)

Based on the information, the process may be:

A) Adiabatic
B) Isobaric
C) Isothermal
D) Isochoric

A

A

This process may be adiabatic. Given that the gas was cooled, it did not maintain constant temperature (Choice C). Isobaric and isovolumetric processes appear are horizontal and vertical lines in pressure-volume graphs, respectively. Adiabatic processes appear hyperbolic on pressure-volume graphs, as described.

163
Q

A reaction has a positive entropy and enthalpy. What can be inferred about the progress of this reaction from this information?

A) The reaction is spontaneous
B) The reaction is nonspontaneous
C) The reaction is at equilibrium
D) There is not enough information to determine whether the reaction is spontaneous or not

A

D

There is not enough information in the problem to determine whether or not the reaction is spontaneous. If the signs of enthalpy and entropy are the same, the reaction is temperature dependent according to ΔG = ΔH - TΔS. Without the temperature, we cannot determine if this reaction is spontaneous, nonspontaneous, or at equilibrium.

164
Q

Pure sodium metal spontaneously combust upon contact with room temperature water. What is true about the equilibrium constant of this combustion reaction at 25 degrees Celsius?

A) Keq < 0
B) 0 < Keq < 1
C) Keq = 1
D) Keq > 1

A

D

Solve this question using the equation ΔGrxn = -RTlnKeq. ΔGrxn is negative (as it must be for a spontaneous reaction), and R and T are always positive. Therefore, ln Keq must also be positive for the sign convention to work out correctly. Since ln (1) = 0, the natural logarithm of any number less than 1 will be negative. In order for ln Keq to be a positive number, Keq must be greater than 1.

165
Q

Which of the following processes has the most exothermic standard heat of combustion?

A) Combustion of ethane
B) Combustion of propane
C) Combustion of n-butane
D) Combustion of n-pentane

A

D

Combustion often involves the reaction of a hydrocarbon with oxygen to produce carbon dioxide and water. Longer hydrocarbon chains yield greater amounts of combustion products and release more heat in the process - that is, the reaction is more exothermic. Of the hydrocarbons listed here, n-pentane is the longest chain.

166
Q

Methanol reacts with acetic acid to form methyl acetate and water.

Type of bond Bond dissociation energy (kJ/mol)
C-C 348
C-H 415
C=O 805
O-H 463
C-O 360

Based on the value in the table, what is the heat of reaction in kJ/mol?

A) 0
B) 464
C) 824
D) 1288

A

A

At first glance, this might seem like a math-heavy problem, but it really doesn’t require any calculations at all. We just have to keep track of which bonds are broken and which bonds are formed. Remember, breaking bonds requires energy, while forming bonds releases energy. Two bonds are broken: a C-O bond between the carbonyl carbon and oxygen of acetic acid, and an O-H bond between the hydroxyl oxygen and hydrogen of methanol. Two bonds are also formed: a C-O bond between the carbonyl carbon and the oxygen of methyl acetate, and an O-H bond between a hydroxyl group and a hydrogen to form water. Given that the same two bonds are broken and formed in this reaction, the energy change must be 0 kJ/mol.

167
Q

At standard temperature and pressure, a chemical process is at equilibrium. What is the free energy of reaction (ΔG) for this process?

A) ΔG > 0
B) ΔG < 0
C) ΔG = 0
D) There is not enough information to determine the free energy of the reaction

A

C

Standard temperature and pressure indicate 0 degrees Celsius and 1 atm. Gibbs free energy is temperature dependent, but if a reaction is at equilibrium, ΔG = 0

168
Q

Consider the chemical reaction in the vessel depicted in the following diagram.
(Reactants = 6 molecules with 2 atoms each
Products = 12 single atom species)

A) The reaction is spontaneous
B) The reaction is non spontaneous
C) The reaction is at equilibrium
D) There is not enough information to determine f the reaction is spontaneous

A

D

There is not enough information available to determine the free energy of this reaction. While the entropy is clearly increasing (there are more particles in the system), it is unclear what the enthalpy change is. Because bonds are breaking, the reaction should be endothermic, meaning that both ΔS and ΔH are positive. In this case, it is a temperature-dependent process, and - without a temperature given - we cannot determine the sign on ΔG.

169
Q

A chemical reaction has a negative enthalpy and a negative entropy. Which of the following terms necessarily describes this reaction?

A) Exothermic
B) Endothermic
C) Exergonic
D) Endergonic

A

A

A reaction with a negative enthalpy is, by definition, exothermic. Because both enthalpy and entropy are negative, this is a temperature-dependent process, and the reaction will be both endergonic and exergonic - but only at particular temperatures.

170
Q

Which of the following statements is true of a process that is spontaneous in the forward direction?

A) ΔG > 0 and Keq > Q
B) ΔG > 0 and Keq < Q
C) ΔG < 0 and Keq > Q
D) ΔG < 0 and Keq < Q

A

C

For a process to progress forward spontaneously, Q must be less than Keq and will therefore have a tendency to move in the direction toward equilibrium. A spontaneous reaction’s free energy is negative by convention.

171
Q

Which of the following devices would be the most appropriate to use to measure the heat capacity of a liquid?

A) Thermometer
B) Calorimeter
C) Barometer
D) Volumetric flask

A

B

A calorimeter measures heat transfer. Although calorimeters often incorporate thermometers, the thermometer itself only tracks temperature, not the heat transfer itself. Choice C is irrelevant; barometers measures changes in pressure. Choice D is also incorrect because volumetric flasks measure quantitie3s of liquid, not the heat capacity of the liquid.

172
Q

Which of the following phase changes is associated with the largest decrease in energy?

A) Fusion
B) Solidification
C) Deposition
D) Sublimation

A

C

Gases have the highest entropy, and solids have the lowest. Therefore, a phase change from a gas to a solid - deposition - would have the largest decrease in entropy of any phase change process.

173
Q

Explosions are necessarily characterized by:

A) ΔG < 0
B) ΔH > 0
C) ΔS < 0
D) T < 0

A

A

In an explosion, a significant amount of heat energy is released, meaning that the reaction is exothermic (ΔH < 0). The entropy change associated with an explosion is positive because energy is dispersed over a much larger area. If this is true, the expression ΔH - TΔS must be negative, indicating that this is an exergonic process (ΔG < 0). Absolute temperature can never be negative.

174
Q

Which of the following sets of conditions would be LEAST likely to result in ideal gas behavior?

A) High pressure and low temperature
B) Low temperature and large volume
C) High pressure and large volume
D) Low pressure and high temperature

A

A

Gases deviate from ideal behavior at higher pressures and lower volumes and temperatures, all of which force molecules closer together. The closer they are, the more they can participate in intermolecular forces, which violates the definition of an ideal gas. At low temperatures, the kinetic energy of the particles is reduced, so collisions with other particles or the wall of the container are more likely to result in significant changes in kinetic energy.

175
Q

What is the density of neon gas in g/L at STP?

A) 452.3
B) 226.0
C) 1.802
D) 0.9018

A

D

Density equals mass divided by volume. The mass of 1 mole of neon gas equals 20.2 grams. At STP, 1 mole of neon occupies 22.4L.

Density = p = mass/volume = 20.2g/22.4L = 10/11 = 1g/L
(actual is 0.902g/L).

176
Q

A leak of helium gas through a small hole occurs at a rate of 3.22 x 10^5mol/s. How will the leakage rates of neon and oxygen gases compare to helium at the same temperature and pressure?

A) Neon will leak faster than helium; oxygen will leak faster than helium
B) Neon will leak faster than helium; oxygen will leak slower than helium
C) Neon will leak slower than helium; oxygen will leak faster than helium
D) Neon will leak slower than helium; oxygen will leak slower than helium

A

D

Graham’s law of effusion states that the relative rates of effusion of two gases at the same temperature and pressure are given by the inverse ratio of the square roots of the masses of the gas particles. In other words, a gas with a higher molar mass will leak more slowly than a gas with a lower molar mass. Both neon and oxygen will leak at slower rates than helium because they both have more mass than helium.

177
Q

Ideal gases:

I. Have no volume
II. Have particles with no attractive forces between them
III. Have no mass

A) I only
B) II only
C) I and II only
D) I, II, and III

A

B

Ideal gases are said to have no attractive forces between molecules. While each particle within the gas is considered to have negligible volume, ideal gases as a whole certainly do have a measurable volume. Gases have molar masses.

178
Q

The kinetic molecular theory states that:

A) The average kinetic energy of a molecule of gas is directly proportional to the temperature of the gas in kelvins
B) Collisions between gas molecules are inelastic
C) Gas particles occupy discrete areas of space
D) All gas molecules have the same kinetic energy at the same temperature

A

A

The average kinetic energy is directly proportional to the temperature of a gas in kelvins. The kinetic molecular theory states that collisions between molecules are elastic and thus do not result in a loss of energy. Gas particles are assumed to take up negligible space in kinetic molecular theory. While the average kinetic energy of any gas as a whole is the same at a given temperature, the particles themselves have a distribution of speeds (as seen in the Maxwell-Boltzmann distribution curve).

179
Q

The gaseous state of matter is characterized by which of the following properties?

I. Gases are compressible
II. Gases assume the volume of their container
III. Gas particles exist as diatomic molecules

A) I only
B) I and II only
C) II and III only
D) I, II, and III

A

B

Gases are easily compressible because they travel freely with large amounts of space between molecules. Because gas particles are far apart from each other and in rapid motion, they tend to take up volume of their container. Many gases exist as diatomic molecules, but this is not a property that characterizes all gases.

180
Q

A gas at a temperature of 27 degrees Celsius has a volume of 60.0mL. What temperature change is needed to increase this gas to a volume of 90.0mL?

A) A reduction of 150 degrees Celsius
B) An increase of 150 degrees Celsius
C) A reduction of 13.5 degrees Celsius
D) An increase of 13.5 degrees Celsius

A

B

We will use Charles’s law. First, we must convert the temperature to kelvins by adding 273 to get 300K as the initial temperature. Think of this as a proportionality: If the volume is multiplied by 3/2, the temperature will also have to be multiplied by 3/2. Thus the final temperature is 450K, which represents a 150K increase (which is equivalent to an increase of 150 degrees Celsius).

181
Q

A gaseous mixture contains nitrogen and helium and has a total pressure of 150 torr. The nitrogen particles comprise 80 percent of the gas, and the helium particles make up the other 20 percent of the gas. What is the pressure exerted by each individual gas?

A) 100 torr nitrogen, 50.0 torr helium
B) 120 torr nitrogen, 30.0 torr helium
C) 30.0 torr nitrogen, 120 torr helium
D) 50.0 torr nitrogen, 100 torr helium

A

B

The partial pressure of each gas is found by multiplying the total pressure by the mole fraction of the gas. Because 80% of the molecules are nitrogen, the mole fraction of nitrogen gas is equal to 0.80. Similarly, for helium, the mole fraction is 0.20. To find the pressure exerted by nitrogen, multiply the total pressure (150 torr) by 0.80 to obtain 120 torr of nitrogen. The remainder, 30 torr, is attributable to helium.

182
Q

In which of the following situations is it impossible to predict how the pressure will change for a gas sample?

A) The gas is cooled at a constant volume
B) The gas is heated at a constant volume
C) The gas is heated, and the volume is simultaneously increased
D) The gas is cooled, and the volume is simultaneously increased

A

C

Both a change in temperature and a change in volume can affect a gas’s pressure. So if one of those two variables is kept constant, we’ll definitely be able to predict which way the pressure will change. At a constant volume, heating the gas will increase its pressure, and cooling the gas will decrease it. What about when both temperature and volume are changing? If both changes have the same effect on pressure, then we can still predict which way it will change. Cooling the has and increasing its volume both decrease pressure. Choice C, on the other hand, presents too vague a scenario for us to predict definitively the change in pressure. Heating the gas would amplify the pressure, while increasing the volume would decrease it. Without knowing the magnitude of each influence, it’s impossible to say whether the pressure would increase, decrease, or stay the same.

183
Q

Experimenters notice that the molar concentration of dissolved oxygen in an enclosed water tank has decreased to one-half its original value. In an attempt to counter this decrease, they quadruple the partial pressure of oxygen in the container. What is the final concentration of the gas?

A) Half of the original concentration
B) The same as the original concentration
C) Double the original concentration
D) Quadruple the original concentration

A

C

Initially the concentration of the gas is decreased to one-half its original value. Recall that concentration (solubility) and partial pressure are directly related - as one increases, the other increases. If the experimenters then quadruple the partial pressure of oxygen in the vessel, the solubility is also increased by a factor of four. One-half times four gives twice the original concentration value. Misreading the answer choices as being related to the concentration before the experimenters increased the partial pressure leads to Choice D.

184
Q

Which phases of solvent and solute can form a solution?

I. Solid solvent, gaseous solute
II. Solid solvent, solid solute
III. Gaseous solvent, gaseous solute

A) I and II only
B) I and III only
C) II and III only
D) I, II, and III

A

D

All three choices can make a solution as long as the two components create a mixture that is of uniform appearance (homogenous). Hydrogen in platinum is an example of a gas in a solid. Brass and steel are examples of homogenous mixtures of solids. The air we breathe is an example of a homogenous mixture of gases; while these are more commonly simply referred to as mixtures, they still fit the criteria of a solution.

185
Q

Two organic liquids, pictured in the figure below, are combined to form a solution. Based on their structures, will the solution closely obey Raoult’s law?

(Benzene and toluene; the latter is exactly the same as benzene but with a CH3, methyl group attached)

A) Yes; the liquids differ due to the additional methyl group on toluene and, therefore, will not deviate from Raoult’s law
B) Yes; the liquids are very similar and, therefore, will not deviate from Raoult’s law
C) No; the liquids differ due to the additional methyl group on toluene and, therefore, will deviate from Raoult’s law
D) No; the liquids both contain benzene rings, which will interact with each other and cause deviation from Raoult’s law

A

B

Benzene and toluene are both organic liquids and have very similar properties. They are both nonpolar and are almost exactly the same size. Raoult’s law states that ideal solution behavior is observed when solute-solute, solvent-solvent, and solute-solvent interactions are all very similar. Therefore, benzene and toluene in solution will be predicted to behave as a nearly ideal solution.

186
Q

Which of the following explanations best describes the mechanism by which solute particles affect the melting point of ice?

A) Melting point is elevated because the kinetic energy of the substance increases
B) Melting point is elevated because the kinetic energy of the substance decreases
C) Melting point is depressed because solute particles interfere with lattice formation
D) Melting point is depressed because solute particles enhance lattice formation

A

C

Melting point depresses upon solute addition. Solute particles interfere with lattice formation, the highly organized state in which solid molecules align themselves. Colder-than-normal conditions are necessary to create the solid structure.

187
Q

The process of formation of a salt solution can be better understood by breaking the process into three steps:

  1. Breaking the solute into its individual components
  2. Making room for the solute in the solvent by overcoming intermolecular forces in the solvent
  3. Allowing solute-solvent interactions to occur to form the solution

Which of the following correctly lists the enthalpy changes for these three steps, respectively?

A) Endothermic, exothermic, endothermic
B) Exothermic, endothermic, endothermic
C) Exothermic, exothermic, endothermic
D) Endothermic, endothermic, exothermic

A

D

The first step will most likely be endothermic because energy is required to break molecules apart. The second step is also endothermic because the intermolecular forces in the solvent must be overcome to allow incorporation of solute particles. The third step will most likely be exothermic because polar water molecules will interact with the dissolved ions, creating a stable solution and releasing energy.

188
Q

The entropy change when a solution forms can be expressed by the term ΔSsoln. When water molecules become ordered around an ion as it dissolves, the ordering would be expected to make a negative contribution to ΔSsoln. An ion that has more charge density will have a greater hydration effect, or ordering of water molecules. Based on this information, which of the following compounds will have the most negative contribution to ΔSsoln?

A) KCl
B) LiF
C) CaS
D) NaCl

A

C

CaS will cause the most negative contribution to ΔSsoln through hydration effects because the Ca2+ and S2- ions have the highest charge density compared to the other ions. All of the other ions have charges of +1 or -1, whereas Ca2+ and S2- each have charges with a magnitude of 2.

189
Q

When ammonia, NH3, is used as a solvent, it can form complex ions. For example, dissolving AgCl in NH3 will result in the complex ion [Ag(NH3)]2+. What effect would the formation of complex ions have on the solubility of a compound like AgCl in NH3?

A) The solubility of AgCl will increase because complex ion formation will cause more ions to exist in solution, which interact with AgCl to cause it to dissociate
B) The solubility of AgCl will increase because complex ion formation will consume Ag+ ions and cause the equilibrium to shift away from solid AgCl
C) The solubility of AgCl will decrease because Ag+ ions are in complexes, and the Ag+ ions that are not complexed will associate with Cl- to form solid AgCl
D) The solubility of AgCl will decrease because complex ion formation will consume Ag+ ions and cause the equilibrium to shift toward the solid AgCl

A

B

Formation of complex ions between silver ions and ammonia will cause more molecules of solid AgCl to dissociate. The equilibrium is driven toward dissociation because the Ag+ ions are essentially being removed from solution when they complex with ammonia. This rationale is based upon Le Châtelier’s principle, stating that when a chemical equilibrium experiences a change in concentration, the system will shift to counteract the change.

190
Q

One hundred grams of sucrose are dissolved in a cup of hot water at 80 degrees Celsius. The cup of water contains 300.00mL of water. What is the percent composition by mass of sugar in the resulting solution? (Note: Sucrose = C12H22O11, density of water at 80 degrees Celsius = 0.975 g/mol)

A) 25.0%
B) 25.5%
C) 33.3%
D) 34.2%

A

B

The mass percent of a solute equals the mass of the solute divided by the mass of the total solution times 100%. Plug in the values given for sucrose, the volume of water and the density of water to determine the mass% of sucrose.

% mass of sucrose
= (100g sucrose)/ [(300mL H2O) (0.975g/mL) +100g Sucrose] x 100%

= 100/(300+100) x 100% = 100/400 x 100% = 25%

Keep in mind that in rounding while calculating, the denominator was estimated to be larger than the actual value, thus giving an answer that is slightly lower than the actual value. Thus the correct answer is 25.5%. Choice A results if rounding error is not taken into account. While these answers are very close, the mass of the water must be slightly less than 300g, given the density value, so the percent composition of sucrose must be slightly higher than 25%. If the solute’s mass is not added to the solvent’s, the calculated value is 34.2%. Choice C neglects both the addition step and the rounding error.

191
Q

Which of the following combinations of liquids would be expected to have a vapor pressure higher than the vapor pressure that would be predicted by Raoult’s law?

A) Ethanol and hexane
B) Acetone and water
C) Isopropanol and methanol
D) Nitric acid and water

A

A

Mixtures that have a higher vapor pressure than predicted by Raoult’s law have stronger solvent-solvent and solute-solute interactions than solvent-solute interactions. Therefore, particles do not want to stay in solution and more readily evaporate, creating a higher vapor pressure than an ideal solution. Two liquids that have different properties, like hexane (hydrophobic) and ethanol (hydrophilic, small) would not have many interactions with each other and would cause positive deviation; i.e. higher vapor pressure. Choices B and C are composed of liquids that are similar to one another and would not show significant deviation from Raoult’s law. Choice D contains two liquids that would interact very well with each other, which would actually cause a negative deviation from Raoult’s law - when attracted to one another, solutes and solvents prefer to stay in liquid form and have a lower vapor pressure than predicted by Raoult’s law.

192
Q

The salt KCl is dissolved in a beaker. To and observer holding the beaker, the solution begins to feel colder as the KCl dissolves. From this observation, one could conclude that:

A) ΔSsoln is large enough to overcome the unfavorable ΔHsoln
B) KCl is mostly insoluble in water
C) ΔSsoln must be negative when KCl dissolves
D) Boiling point depression will occur in this solution

A

A

Dissolution is governed by enthalpy and entropy, which are related by the equation ΔGsoln = ΔHsoln - TΔSsoln. The cooling of the solution indicates that heat is used up in this bond-breaking reaction. In other words, dissolution is endothermic, and ΔH is positive. The reaction is occurring spontaneously, so ΔG must be negative. The only way that a positive ΔH can result in a negative ΔG is if the entropy, ΔS, is a large, positive value. Conceptually, that means that the only way the solid can dissolve is if the increase in entropy is great enough to overcome the increase in enthalpy. Choice B is incorrect because it is clearly stated in the question stem that KCl dissolves; further, all salts of Group 1 metals are soluble. Choice C is incorrect because ΔSsoln must be positive in order for KCl to dissolve. Finally, Choice D is incorrect because solute dissolution would cause the boiling point to elevate, not depress. It is also not a piece of evidence that could be found simply by observing the beaker’s temperature change.

193
Q

A saturated solution of cobalt (III) hydroxide (Ksp = 1.6 x 10^-44) is added to a saturated solution of thallium (III) hydroxide (Ksp = 6.3 x 10^-46). What is likely to occur?

A) Both cobalt (III) hydroxide and thallium (III) hydroxide remain stable in solution
B) Cobalt (III) hydroxide precipitates and thallium (III) hydroxide remains stable in solution
C) Thallium (III) hydroxide precipitates and cobalt (III) hydroxide remains stable in solution
D) Both thallium (III) hydroxide and cobalt (III) hydroxide precipitate

A

D

Since both salts have a formula MX3 (one of one particle, three of another), it is possible to directly compare the molar solubilities of each. When the solutions are mixed, [OH-] is above saturation levels for both the cobalt and the thallium in the solution. Since thallium (III) hydroxide has a smaller Ksp than that of cobalt (III) hydroxide, it will react first. The ion product of the mixed solution is higher than the Ksp for thallium (III) hydroxide, and the system will shift left to precipitate solid thallium (III) hydroxide. After the thallium (III) hydroxide precipitates, a small excess of [OH-] will remain, which gives an ion product slightly above the Ksp of cobalt (III) hydroxide. This will cause a small amount (1%-3%) of cobalt (III) hydroxide to also precipitate.

194
Q

Which of the following is not a Bronsted-Lowry base?

A)   H
       I
      N
    /    \
  H     H

B) F-

C) O
/ \
H H

D) H - O - N = O

A

D

A Bronsted-Lowry base is defined as a proton acceptor. Ammonia, fluoride, and water each accept a proton. HNO2, is a far better Bronsted-Lowry acid, donating a proton to solution.

195
Q

Which of the following is closest to the pH of a solution containing 5mM H2SO4?

A) 1
B) 2
C) 3
D) 4

A

B

First, convert the concentration to 5 x 10^-3 M. Next, because sulfuric acid is a strong acid, we can assume that, for the majority of sulfuric acid molecules (although not all all), both protons will dissociate. The concentration of hydrogen ions is therefore 2 x (5 x 10^-3), or 10^-2. The equation for pH is pH = -log{H+]. If [H+} = 10^-2 M, then pH = 2

196
Q

Which of the following represents chloric acid?

A) HClO3
B) ClO3-
C) HClO2
D) HClO

A

A

Answering this question is simply a matter of knowing nomenclature. Acids ending in -ic are derivatives of anions ending in -ate, while acids ending in -ous are derivatives of anions ending in -ite. ClO3- is chlorate because it has more oxygen than the other commonly occurring ion, ClO2-, which is named chlorite. Therefore, HClO3 is chloric acid. HClO2 represents chlorous acid. HClO represents hypochlorous acid.

197
Q

Which of the following bases is the weakest?

A) KOH
B) NH3
C) CH3NH2
D) Ca(OH)2

A

B

Soluble hydroxides of Group IA and IIA metals are strong bases. Choices B and C are both weak bases; however, methylamine contains an alkyl group, which is electron-donating. This increases the electron density on the nitrogen in methylamine, making it a stronger (Lewis) base. Therefore, ammonia is the weakest base.

198
Q

The function of a buffer is to:

A) Maintain a neutral pH
B) Resist changes in pH when small amounts of acid or base are added
C) Slow down reactions between acids and bases
D) Speed up reactions between acids and bases

A

B

The purpose of a buffer is to resist changes in the pH of a reaction. Buffers are not generally used to affect the kinetics of a reaction. Choice A is correct only in specific circumstances where the pH of the buffer solution itself is neutral. Many natural buffer systems maintain pH in the acidic or basic ranges.

199
Q

What is the pH of a solution with an ammonium concentration of 70mM and an ammonia concentration of 712mM? (Note: The pKb of ammonia is 3.45).

A) 2.45
B) 4.45
C) 9.55
D) 11.55

A

D

The question is asking for pH, but because of the information given, we must first find the pOH and then subtract it from 14 to get the pH. Use the Henderson-Hasselbalch equation:

pOH = pKb + log([conjugate acid] / [base])

= 3.45 + log(70mM / 712mM)

= 3.45 + log(1 / 10) = 3.45 - 1 = 2.45

If the pOH = 2.45, then the pH = 14 - 2.45 = 11.55

200
Q

What is the gram equivalent weight of phosphoric acid?

A) 24.5g
B) 32.7g
C) 49.0g
D) 98.0g

A

B

Gram equivalent weight is the weight (in grams) that releases 1 acid or base equivalent from a compound. Because H3PO4 contains 3 protons, we find the gram equivalent weight by dividing the mass of one mole of the species by 3. The molar mass of phosphoric acid is 98g/mol, so the gram equivalent weight is 32.7g.

201
Q

What is the {H3O+] of a 2M aqueous solution of a weak acid HXO2 with Ka = 3.2 x 10^-5?

A) 6.4 x 10^-5 M
B) 1.3 x 10^-4 M
C) 4.0 x 10^-3 M
D) 8.0 x 10^-3 M

A

This question requires application of the acid dissociation constant. Weak acids do not dissociate completely; therefore, all three species that appear in the balanced equation will be present in solution. Hydrogen ions and conjugate base anions dissociate in equal amounts, so [H+] = [XO2-]. If the initial concentration of HXO2 was 2M and some amount x dissociates, we will have x amount of H3O+ and XO2- at equilibrium, with 2M - x amount of HXO2 at equilibrium.

Note that x was considered negligible when added or subtracted, per usual. Solving for x, we get:

x^2/2 = 3.2 x 10^-5 -> x^2 = 6.4 x 10^-5

= 64 x 10^-6 = 8 x 10^-3

202
Q

A solution is prepared with an unknown concentration of a theoretical compound with a Ka of exactly 1.0. What is the pH of this solution?

A) Higher than 7
B) Exactly 7
C) Less than 7
D) There is not enough information to answer the question

A

C

A higher Ka implies a stronger acid. Weak acids usually have a Ka that is several orders of magnitude below 1. The pKa of a compound is the pH at which there are equal concentrations of acid and conjugate base; the pKa of this compound would be -log 1 = 0. With such a low pKa, this compound must be an acid. Therefore, the pH of any concentration of this compound must be below 7.

203
Q

Which of the following is NOT a characteristic of an amphoteric species?

A) Amphoteric species can act as an acid or a base, depending on its environment
B) Amphoteric species can act as an oxidizing or reducing agent, depending on its environment
C) Amphoteric species are sometimes amphiprotic
D) Amphoteric species are always nonpolar

A

D

An amphoteric species is one that can act either as an acid or a base, depending on its environment. Proton transfers are classic oxidation-reduction reactions. Choice C is true because many amphoteric species, such as water and bicarbonate, can either donate or accept a proton. Choice D is false, and thus the correct answer because amphoteric species can be either polar or nonpolar in nature.

204
Q

What is the approximate pH of a 1.2 x 10^-5 M aqueous solution of NaOH?

A) 4.92
B) 7.50
C) 9.08
D) 12.45

A

C

NaOH is a strong base; as such, there will be 1.2 x 10^-5 M OH- in solution. Based on this information alone, the pOH must be between 4 and 5, and the pH must be between 9 and 10.

Using the shortcut, pOH = 5 - 0.12 = 4.88.

pH = 14 - pOH = 14 - 4.88 = 9.12 (actual 9.08).

205
Q

How many liters of 2 M Ba(OH)2 are needed to titrate a 4L solution of 6 M H3PO4?

A) 1.33L
B) 12L
C) 18L
D) 56L

A

C

Use the equivalence point equation: NaVa = NbVb. Ba(OH)2 can dissociate to give two hydroxide ions, so its normality is 2 M x 2 = 4. H3PO4 can dissociate to give three hydronium ions, so its normality is 6 M x 3 = 18N. Plugging into the equation, we get (18N)(4L) = (4N)(Vb). Therefore, Vb is 18L.

206
Q

Consider the following equation:

6 Na (s) + 2 NH3 (aq) -> 2 Na3N (s) + 3 H2 (g)

What species acts as an oxidizing agent?

A) Na
B) N in NH3
C) H in NH3
D) H2

A

C

The oxidizing agent is the species that is reduced in any given equation. In this problem, six hydrogen atoms with +1 oxidation states in NH3 are reduced to three neutral H2 molecules.

207
Q

How many electrons are involved in the following half-reaction after it is balanced?

Cr2O7^2- + H+ + e- -> Cr2+ + H2O

A) 2
B) 8
C) 12
D) 16

A

B

First, balance the atoms in the equation:

Cr2O7^2- + 14H+ -> 2Cr2+ + 7H2O

Now, adjust the number of electrons to balance the charge. Currently, the left side has a charge of +12 (-2 from dichromate and +14 from protons). The right side has a charge of +4 (+2 from each chromium cation). To decrease the charge on the left side from +12 to +4, we should add 8 electrons:

CrO7^2- + 14H+ + 8e- -> 2Cr2+ + 7H2O

208
Q

Lithium aluminum hydride (LiAlH4) is often used in laboratories because of its tendency to donate a hydride ion. Which of the following roles would lithium aluminum hydride likely play in a reaction?

A) Strong reducing agent only
B) Strong oxidizing agent only
C) Both a strong reducing agent and strong oxidizing agent
D) Neither a strong reducing agent nor a strong oxidizing agent

A

A

Hydride ions are composed of a hydrogen nucleus with two electrons, thereby giving it a negative charge and a considerably tendency to donate electrons. LiAlH4 is therefore a strong reducing agent. Strong reducing agents tend to have metals or hydrides; strong oxidizing agents tend to have oxygen or a similarly electronegative element.

209
Q

What is the oxidation number of chlorine in NaClO?

A) -1
B) 0
C) +1
D) +2

A

C

In NaClO (sodium hypochlorite), sodium carries its typical +1 charge, and oxygen carries its typical +2 charge. This means that the chlorite atom must carry a +1 charge in order to balance the overall charge of zero.

210
Q

Which of the following is the correct net ionic reaction for the reaction of copper with silver(I) nitrate?

A) Cu + AgNO3 -> Cu(NO3)2 + Ag
B) Cu + 2Ag+ + 2NO3- -> Cu2+ + 2NO3- + 2Ag
C) 2Ag+ + 2NO3- -> 2NO3- + 2Ag+
D) Cu + 2Ag+ -> Cu2+ + 2Ag

A

D

A net ionic equation represents each of the aqueous ions comprising the reactants and products as individual ions, instead of combining them as formula units. Thus, Choice A is not a net ionic reaction. The term net means that the correct answer does not include any spectator ions (ions that do not participate in the reaction). In this reaction, nitrate (NO3-) remains unchanged. Therefore, Choices B and C are incorrect.

211
Q

One way to test for the presence of iron in solution is by adding potassium thiocyanate to the solution. The product when this reagent reacts with iron is FeSCN2+, which creates a dark red color in solution via the following new ionic equation:

Fe3+ + SCN- -> FeSCN2+

How many grams of iron sulfate would be needed to produce 2 moles of FeSCN2+?

A) 110g
B) 220g
C) 400g
D) 500g

A

C

What you are shown is a new ionic equation. If two moles of FeSCN are created, two moles of Fe3+ must be used because the mole ratio is 1:1. Iron sulfate has the formula Fe2(SO4)3 because sulfate has a charge of -2 and iron has a charge of +3 (based on the new ionic equation). Therefore, one mole of iron sulfate is needed to make two moles of iron for the reaction. The molar mass of iron sulfate is

2 x 55.8g/mol + 3 x 32.1g/mol + 12 x 16.0g/mol

=399.9g/mol

This most closely matches Choice C. The most common error would be to calculate the amount of iron, which would be 111.6g.

212
Q

During the assigning of oxidation numbers, which of the following elements would most likely be determined last?

A) Ar
B) F
C) Sr
D) Ir

A

D

When assigning oxidation numbers, one starts with elements of known oxidation state first, and determines the oxidation state of the other elements by deduction. As a noble gas, argon, will always have an oxidation state of 0. As a Group VIIA element, fluorine, will always have an oxidation state of 0 (by itself) or -1 (in a compound). As a Group IIA element, strontium will have an oxidation state of 0 (by itself) or +2 (in a compound). Like most transition metals, iridium, can have various oxidation states, ranging from -3 to +8. Therefore, one would have to determine the oxidation states of other atoms in an iridium-containing compound to determine iridium’s oxidation number.

213
Q

As methanol is converted to methanal, and then methanoic acid, the oxidation number of the carbon:

A) Increases
B) Decreases
C) Increases, then decreases
D) Decreases, then increases

A

A

The formula for methanol is H3COH, for methanal is HCHO, and for methanoic acid is HCOOH. If we assign oxidation numbers to carbon in each molecule, it starts at -2, then becomes 0, then becomes +2. In general, it is often easier to think of oxidation as a gain of bonds to oxygen (or a similarly electronegative element) or loss of bonds to hydrogen for organic compounds. Therefore, because the carbon is oxidized as one converts from an alcohol to an aldehyde to a carboxylic acid, the oxidation number must increase.

214
Q

In the compound KH2PO4, which element has the highest oxidation number?

A) K
B) H
C) P
D) O

A

C

Start with the atoms that have oxidation states of which you are certain. Potassium is a Group IA metal, and therefore must have an oxidation state of +1. Hydrogen is almost always +1, unless it is paired with a less electronegative element (which is not the case here). Oxygen is generally -2. Because there are four oxygens, they create a total negative charge of -8 which is partially balanced by two hydrogens (+2) and potassium (+1). Therefore, phosphorus has a +5 charge, making it the highest oxidation state.

215
Q

If a certain metal has multiple oxidation states, its acidity as an oxide generally increases as the oxidation state increases. Therefore, which of the following tungsten compounds is likely to be the strongest acid?

A) WO2
B) WO3
C) W2O3
D) W2O5

A

B

Recall that oxygen has an oxidation state of -2. Therefore, in tungsten(IV) oxide, Choice A, tungsten has an oxidation state of +4. In tungsten(VI), Choice B, it has an oxidation state of +6. In tungsten(III), Choice C, it is +3. In tungsten pentoxide, Choice D, it is +5.

216
Q

Consider the following steps in the reaction between oxalic acid and chlorine:

I. Cl2 + H2O -> HOCl + Cl- + H+
II. H2C2O4 -> H+ + HC2O4-
III. HOCl + HC2O4- -> H2O + Cl- + 2CO2

Which of these steps, occurring in aqueous solution, is an example of a disproportionation reaction?

A) I only
B) III only
C) I and III only
D) I, II, and III

A

A

Step I is a disproportionation reaction because chlorine starts with an oxidation state of 0 in the reactants and ends up with an oxidation state of +1 in HOCl and -1 as Cl-. In the other reactions, no element appears with different oxidation states in two different products. Therefore, only Step I is a disproportionation reaction.

217
Q

Potentiometry in an oxidation-reduction titration is analogous to performing an acid-base titration with a(n):

A) Acidic indicator
B) Basic indicator
C) pH meter
D) Oxidizing agent

A

C

Potentiometry refers to carrying out an oxidation-reduction titration with a voltmeter present to get precise readings of the reactions electromotive force (emf) to determine the endpoint. This is analogous to using a pH meter in an acid-base titration because it uses technology to get precise readings for plotting a titration curve. Indicators can be used in both acid-base and redox titrations, but provide a qualitative (rather than quantitative) analysis of the titration. Oxidizing and reducing agents are used in redox titrations, not acid-base titrations.

218
Q

Rusting occurs due to the oxidation-reduction reaction of iron with environmental oxygen:

4 Fe (s) + 3 O2 (g) -> 2 Fe2O3 (s)

Some metals cannot react with oxygen in this fashion. Which of the following best explains why iron can?

A) Iron has a more positive reduction potential than those metals, making it more likely to donate electrons to oxygen
B) Iron has a more positive reduction potential than those metals, making it more likely to accept electrons from oxygen
C) Iron has a less positive reduction potential than those metals, making it more likely to donate electrons to oxygen
D) Iron has a less positive reduction potential than those metals, making it more likely to accept electrons from oxygen

A

C

In the oxidation-reduction reaction of a metal with oxygen, the metal will be oxidized (donate electrons) and oxygen will be reduced (accept electrons). A species with a higher reduction potential is more likely to be reduced, and a species with a lower reduction potential is more likely to be oxidized. Based on the information in the question, iron is oxidized more readily than those metals; this means that iron has a lower reduction potential.

219
Q

Given the following standard reduction potentials:

Zn^2+ + 2e- -> Zn Ered = -0.763V
Ag+ + e- -> Ag Ered = +0.337V

What is the standard electromotive force of the following reaction?

Zn^2+ + 2Ag -> 2Ag+ + Zn

A) -2.2V
B) -1.1V
C) +1.1V
D) +2.2V

A

B

To determine the standard electromotive force of a cell, simply subtract the standard reduction potentials of the two electrode. In this case, the cathode is zinc because it is being reduced; the anode is silver because it is being oxidized. Thus,

Ecell = Ered(cathode) - Ered(anode)

= -0.763 - 0.337 = -1.10V

While we must multiply the silver half-reaction by two to balance electrons, the actual value for the reduction potential does not change. Remember that the standard reduction potential is determined by the identity of the electrode, not the amount of it present.

220
Q

Consider the following data:

Hg^2+ + 2e- -> Hg Ered = +0.85V
Cu+ + e- -> Cu Ered = +0.52V
Zn^2+ + 2e- -> Zn Ered = -0.76V
Al^3+ + 3e- -> Al Ered = -1.66V

The anode of a certain galvanic cell is composed of copper. Which of the metals from the data table can be used at the cathode, assuming equal concentrations of the two electrolyte solutions?

A) Hg
B) Cu
C) Zn
D) Al

A

A

Oxidation occurs at the anode, and reduction occurs at the cathode. Because Cu is the anode, it must be oxidized. The reduction potential of the cathode cannot be less than that of the anode for a galvanic cell. Therefore, mercury must be the cathode. In a concentration cell, the same material is used as both the cathode and anode; however, this question assumes equal concentrations. If both electrolyte solutions have the same concentration, there will be no oxidation-reduction reaction and, therefore, no anode or cathode.

221
Q

An electrolytic cell is filled with water. Which of the following will move toward the cathode of such a cell?

I. H+ ions
II. O^2- ions
III. Electrons

A) I only
B) II only
C) I and III only
D) II and III only

A

C

In an electrolytic cell, ionic compounds are broken up into their constituents; the cations (positively charged ions) migrate toward the cathode, and the anions (negatively charged ions) migrate toward the anode. In this case, the cations are H+ ions (protons), so option I is correct. Electrons flow from anode to cathode in all types of cells, meaning that option III is also correct. Option II is incorrect for two reasons. First, it is unlikely that the anions in any cell would be O^2- rather than OH-. Second, and more significantly, these anions would flow to the anode, not the cathode.

222
Q

If the value of Ecell is known, what other data is needed to calculate ΔG?

A) Equilibrium constant
B) Reaction quotient
C) Temperature of the system
D) Half-reactions of the cells

A

D

This answer comes directly from the equation relating Gibbs free energy and Ecell. ΔG = -nFEcell, where n is the number of moles of electrons transferred and F is the Faraday constant, 96,485 C/mol e-. To determine n, one must look at the balanced half-reactions occurring in the oxidation-reduction reaction.

223
Q

Which of the following compounds is LEAST likely to be found in the salt bridge of a galvanic cell?

A) NaCl
B) SO3
C) MgSO3
D) NH4NO3

A

B

Salt bridges contain inert electrolytes. Ionic compounds such as Choices A, C, and D, are known to be strong electrolytes because they completely dissociate in solution. Choice B cannot be considered an electrolyte because its atoms are covalently bonded and will not dissociate in aqueous solution. Choices B and C may appear similar, but there is an important distinction to be made. Choice C implies that Mg^2+ and SO3^2- are the final, dissociated ionic constituents, while Choice B implies that neutral SO3 would have to be dissolved in solution.

224
Q

If the surface area of electrode material in an electrochemical cell is tripled, what else is necessarily tripled?

I. Ecell
II. Current
III. Keq

A) I only
B) II only
C) I and II only
D) II and III only

A

B

Potential, as measured by Ecell, is dependent only on the identity of the electrodes and not the amount present. Similarly, the equilibrium constant depends on only the identity of the electrolyte solutions and the temperature. However, as the electrode material is increased, the surface area participating in oxidation-reduction reactions is increased and more electrons are released, making statement II correct.

225
Q

Which of the following can alter the emf of an electrochemical cell?

A) The mass of the electrodes
B) The length of the wire connecting the half-cells
C) The overall size of the battery
D) The temperature of the solutions in the half-cells

A

D

Ecell is dependent upon the change in free energy of the system through the equation RT ln Keq = nFEcell. The temperature, T, appears in this equation: thus, a change in temperature will impact the Ecell.

226
Q

Which of the following statements could be true about a Na-Cd cell, based on the information below?

Na+ + e- -> Na Ered = -2.71V
Cd^2+ + 2e- -> Cd Ered = -0.40V

A) It is a galvanic cell, and sodium is the cathode
B) It is an electrolytic cell, and cadmium is the anode
C) It is a galvanic cell, with Ecell = 3.11V
D) It is an electrolytic cell, with Ecell = -3.11V

A

B

If this were a galvanic cell, the species with the more positive reduction potential (cadmium) would be reduced. The cathode is always reduced in an electrochemical cell, so sodium could not be the cathode in such a galvanic cell. Sodium would be the cathode in an electrolytic cell, however, which would make cadmium the anode. Note that we do not have to determine Ecell because we already know the answer. However, the Ecell would be -2.71 - (-0,40) - -2.31V for an electrolytic cell and +2.31V for a galvanic cell.

227
Q

Which of the following choices is indicative of a spontaneous reaction, assuming standard conditions?

A) Ecell is negative
B) Q = Keq
C) The cell is a concentration cell
D) Keq > 1

A

D

A spontaneous electrochemical reaction has a negative ΔG. Using the equation ΔG = -RT ln Keq, Keq > 1 would result in ln Keq > 0, which means ΔG < 0. A negative electromotive force or equilibrium state would not correspond to a spontaneous reaction. Concentration cells can be spontaneous; however, if the concentration cell has reached equilibrium, it would cease to be a spontaneous reaction. When an answer choice may be true, but does not have to be - it is the wrong answer on Test Day.

228
Q

For a cell with the following half-reactions:

Anode
SO2 + 2H2O -> SO4^2- + 4H+ + 2e-

Cathode
Pd^2+ + 2e- -> Pd

How would decreasing the pH of the solution inside the cell affect the electromotive force (emf)?

A) The emf would decrease
B) The emf would remain the same
C) The emf would increase
D) The emf would become zero

A

A

A change in pH has a direct correlation to the hydrogen ion (H+) concentration. Decreasing the pH increases the H+ concentration, which means the concentration of products has increased in the oxidation of sulfur dioxide. This means it would be harder to liberate electrons, thus decreasing the emf. One could also view this decrease in oxidation potential as an increase in reduction potential. If Ered(anode) increases, then Ecell must decrease according to Ecell = Ered(cathode) - Ered(anode).

229
Q

An electrolytic cell necessarily has:

A) ΔS > 0
B) ΔG < 0
C) Keq < 1
D) Ecell > 0

A

C

An electrolytic cell is nonspontaneous. Therefore, the ΔG must be positive and Ecell must be negative. The change in entropy may be positive or negative, depending on the species involved. According to the equation ΔG = -RT ln Keq, Keq < 1 would result in ln Keq < 0, which means ΔG > 0.

230
Q

Which of the following is the best explanation for the fact that a larger mass of electrodes are required for lead-acid batteries, as compared to other batteries, to produce a certain current?

A) The lead-acid electrolyte, sulfuric acid, is diprotic and incompletely dissociates in solution
B) The energy density of lead-acid electrodes is higher than that of other batteries
C) The electrolytes in other batteries less readily dissociate than those of lead-acid batteries
D) The energy density of lead-acid electrodes is lower than that of other batteries

A

D

Compared to other cell types, lead-acid batteries have a characteristically low energy density. While Choice A is a true statement, the incomplete dissociation of sulfuric acid does not fully explain the low energy density of lead-acid batteries. Choice C is likely to be an opposite; the more easily the electrodes dissociate, the easier it is to carry out oxidation-reduction reaction with them.

231
Q

Which of the following best describes why over-charging a Ni-Cd battery is not detrimental?

A) The energy density of a Ni-Cd battery is high, so it can store more charge than other batteries per its mass
B) The electrodes of a Ni-Cd battery can discharge through the circuit when they are fully charged
C) The Ni-Cd battery will stop accepting electrons from an outside source when its electrodes are recharged
D) Ni-Cd batteries have a high surge current and can dissipate the overcharge before damage can occur to electrodes

A

C

During the recharge cycle, Ni-Cd cells will accept current from an outside source until the Cd and NiO(OH) electrodes are pure; at this point, the reaction will stop because Cd(OH)2 runs out and no more electrons can be accepted. Choices A and B are both true statements, but they fail to explain why overcharging the battery (continuing to try to run current into the battery even when the electrodes are reverted to their original state) is not a problem with Ni-Cd batteries. Finally, surge current refers to the initial burst of current seen in some batteries; once charged, the surge current will not increase even if the power sources continues to be run because no additional charge will be stored on the electrodes.